OB Final Quiz/Exam

¡Supera tus tareas y exámenes ahora con Quizwiz!

A nurse is assessing a pregnant woman who has come to the clinic. The woman reports that she feels some heaviness in her thighs since yesterday. The nurse suspects that the woman may be experiencing preterm labor based on which additional assessment findings? Select all that apply. Answers: A. malodorous vaginal discharge B. dysuria C. dull low backache D. four to five contractions in 1 hour E. constipation

??? Symptoms of preterm labor are often subtle and may include change or increase in vaginal discharge with mucus, water, or blood in it; pelvic pressure; low, dull backache; nausea, vomiting or diarrhea, and heaviness or aching in the thighs. Constipation is not known to be a sign of preterm labor. Preterm labor is assessed when there are more than six contractions per hour. (ch. 21)

The nurse is reviewing the medical record of a newborn born 2 hours ago. The nurse notes that the newborn was delivered at 35 weeks' gestation. How would the nurse classify this newborn? Answers: A. full term B. late preterm C. preterm D. post-term

A late preterm infant is one born between 34 to 36 weeks of gestation. A preterm infant is one born before 37 completed weeks' gestation. A full-term infant is one born between 38 to 41 weeks' gestation. A post-term newborn is one born at 42 weeks' gestation or later. (ch. 23)

A woman is being evaluated for pelvic organ prolapse. A postvoid residual urine specimen is obtained via a catheter. Which residual volume finding would lead the nurse to suspect the need for further testing? Answers: A. 120 mL B. 75 mL C. 95 mL D. 50 mL

A. 120 mL A postvoid residual urine specimen of greater than 100 mL indicates the need for further urodynamic evaluation and testing. (ch. 7)

A nurse is providing care to a female client receiving treatment for a Bartholin's cyst. The client has had a small loop of plastic tubing secured in place to allow for drainage. The nurse instructs the client that she will have a follow-up appointment for removal of the plastic tubing at which time? Answers: A. 3 weeks B. 4 weeks C. 2 weeks D. 1 week

A. 3 weeks The follow-up visit for removal of the plastic tubing is in approximately 3 weeks. After the Word catheter is inserted, the balloon tip is inflated, and it is left in place for 4 to 6 weeks. (ch. 7)

A woman is receiving magnesium sulfate as part of her treatment for severe preeclampsia. The nurse is monitoring the woman's serum magnesium levels. Which level would the nurse identify as therapeutic? Answers: A. 6.1 mEq/L B. 8.4 mEq/L C. 3.3 mEq/L D. 10.8 mEq/L

A. 6.1 mEq/L Although exact levels may vary among agencies, serum magnesium levels ranging from 4 to 7 mEq/L are considered therapeutic, whereas levels more than 8 mEq/dL are generally considered toxic. (ch. 19)

When describing programs for breast cancer screening, the nurse includes breast self-examination (BSE). Which statement most accurately reflects the current thinking about breast self-examination? Answers: A. BSE is essential for early breast cancer detection. B. A woman performing BSE has breast awareness. C. BSE plays a minimal role in detecting breast cancer. D. A clinical breast exam has replaced BSE.

A. A woman performing BSE has breast awareness Breast self-examination (BSE) is a technique that enables a woman to detect any changes in her breasts. Breast self-exams, once thought essential for early breast cancer detection, are now considered optional. Instead, breast awareness is stressed. Breast awareness refers to a woman being familiar with the normal consistency of both breasts and the underlying tissue. This emphasis is now on awareness of breast changes, not just discovery of cancer. Research has shown that breast self-examination plays a small role in detecting breast cancer compared with self-awareness. However, doing breast self-examination is one way for a woman to know how her breasts normally feel so that she can notice any changes that do occur. Clinical breast examination has not replaced BSE. (ch. 6)

A nurse is reading a journal article about the trends in health care and health care delivery over the past several centuries. When reading about the practices during the 18th century, which information would the nurse most likely find? Answers: A. absence of men attending childbirth B. epidemics of puerperal fever C. use of physicians for birth D. use of twilight sleep for childbirth

A. Absence of men attending childbirth During the 18th century (1700s), men did not attend childbirth because it was thought to be indecent. The use of physicians for birth occurred during the 19th century, which was also the time of puerperal fever epidemics. Twilight sleep was used in the 20th century. (ch. 1)

The nurse is assessing a newborn who is large for gestational age. The newborn was born breech. The nurse suspects that the newborn may have experienced trauma to the upper brachial plexus based on which assessment finding? Answers: A. absent Moro reflex B. hand weakness C. absent grasp reflex D. facial asymmetry

A. Absent moro reflex An injury to the upper brachial plexus, or Erb's palsy, is manifested by adduction, pronation, and internal rotation of the affected extremity, absent shoulder movement, absent Moro reflex and positive grasp reflex. An absent grasp reflex and hand weakness is noted with a lower brachial plexus injury. Facial asymmetry is associated with a cranial nerve injury. (ch. 24)

A pregnant woman admitted to the labor and birth suite undergoes rapid HIV testing and is found to be HIV-positive. Which action would the nurse expect to include when developing a plan of care for this woman? Select all that apply. Answers: A. avoidance of scalp electrodes for fetal monitoring B. administration of penicillin G at the onset of labor C. electing for the use of forceps-assisted delivery D. administering zidovudine at the onset of labor. E. refraining from obtaining fetal scalp blood for pH testing

A. Avoidance of scalp electrodes for fetal monitoring D. Administering zidovidine at the onset of labor E. Refraining from obtaining fetal scalp blood for pH testing To reduce perinatal transmission, HIV-positive women are given zidovudine (2 mg/kg IV over an hour, and then a maintenance infusion of 1 mg/kg per hour until birth) or a single 200-mg oral dose of nevirapine at the onset of labor; the newborn is given ZDV orally (2 mg/kg body weight every 6 hours) and should be continued for 6 weeks (Gardner, Carter, Enzman-Hines, & Hernandez, 2011). To further reduce the risk of perinatal transmission, ACOG and the U.S. Public Health Service recommend that HIV-infected women with plasma viral loads of more than 1,000 copies per milliliter be counseled regarding the benefits of elective cesarean birth (Reshi & Lone, 2010). Additional interventions to reduce the transmission risk would include avoiding use of scalp electrode for fetal monitoring or doing a scalp blood sampling for fetal pH, delaying amniotomy, encouraging formula feeding after birth, and avoiding invasive procedures such as forceps or vacuum-assisted devices. (ch. 14)

A nurse is assessing a newborn's reflexes. The nurse strokes the lateral sole of the newborn's foot from the heel to the ball of the foot to elicit which reflex? Answers: A. Babinski B. plantar grasp C. tonic neck D. stepping

A. Babinski The Babinski reflex is elicited by stroking the lateral sole of the newborn's foot from the heel toward and across the ball of the foot. The tonic neck reflex is tested by having the newborn lie on his back and then turn his head to one side. The stepping reflex is elicited by holding the newborn upright and inclined forward with the soles of the feet on a flat surface. The plantar grasp reflex is elicited by placing a finger against the area just below the newborn's toes. (ch. 18)

When the nurse is assessing a postpartum client approximately 6 hours after delivery, which finding would warrant further investigation? Answers: A. blood pressure 90/50 mm Hg B. profuse sweating C. deep red, fleshy-smelling lochia D. voiding of 350 cc

A. Blood pressure 90/50 mm Hg In most instances of postpartum hemorrhage, blood pressure and cardiac output remain increased because of the compensatory increase in heart rate. Thus, a decrease in blood pressure and cardiac output are not expected changes during the postpartum period. Early identification is essential to ensure prompt intervention. Deep red, fleshy-smelling lochia is a normal finding 6 hours postpartum. Voiding in small amounts such as less than 150 cc would indicate a problem, but 350 cc would be appropriate. Profuse sweating also is normal during the postpartum period. (ch. 15)

A client states, "I think my waters broke! I felt this gush of fluid between my legs." The nurse tests the fluid with a Nitrazine swab and confirms membrane rupture if the swab turns: Answers: A. blue. B. olive green. C. yellow. D. pink.

A. Blue Amniotic fluid is alkaline and turns a Nitrazine swab blue. Nitrazine swabs that remain yellow to olive green suggests that the membranes are most likely intact. (ch. 14)

A breast biopsy indicates the presence of malignant cells, and the client is scheduled for a mastectomy. When preparing the client's preoperative plan of care, which area would the nurse most likely address as the priority? Answers: A. body image B. fluid balance C. urinary elimination D. activity

A. Body image The diagnosis of breast cancer and subsequent removal of the breast via surgery can affect all aspects of life for the woman, but most significantly her body image due to the loss of a body part. Therefore, the nurse would need to address body image as the priority. Issues involving fluid volume, activity, and urinary elimination are possible due to the effects of surgery, but these are not as important as the client's body image. (ch. 6)

A nurse is reviewing the medical record of a pregnant client. The nurse suspects that the client may be at risk for dystocia based on which factor? Answers: A. breech fetal presentation B. multiparity C. short maternal stature D. plan for pudendal block anesthetic use E. maternal age over 35

A. Breech fetal presentation C. Short maternal stature E. Maternal age over 35 According to American College of Obstetrics and Gynecology, factors associated with an increased risk for dystocia include epidural analgesia, excessive analgesia, multiple gestation, hydramnios, maternal exhaustion, ineffective maternal pushing technique, occiput posterior position, longer first stage of labor, nulliparity, short maternal stature (less than 5 feet tall), fetal birth weight (more than 8.8 lb 3.9 kg), shoulder dystocia, abnormal fetal presentation or position (breech), fetal anomalies (hydrocephalus), maternal age older than 35 years, high caffeine intake, overweight, gestational age more than 41 weeks, chorioamnionitis, ineffective uterine contractions, and high fetal station at complete cervical dilation. (ch. 21)

When preparing the discharge teaching plan for the woman who had surgery to correct pelvic organ prolapse, which information would the nurse include? Answers: A. care of the indwelling catheter at home B. daily douching with dilute vinegar solution C. emphasis on coughing to prevent complications D. return to usual activity level in a few days

A. Care of the indwelling catheter at home Following surgery to repair a pelvic organ prolapse, the nurse would teach the woman about caring for the indwelling catheter, which will remain in place for approximately 1 week. Activities that increase intra-abdominal pressure, such as straining, sneezing, or coughing, should be avoided. The woman also should avoid heavy lifting or straining for several weeks. Pelvic rest is prescribed until the operative area is healed in 6 weeks. Douching is indicated if the woman had a pessary inserted, not surgery. (ch. 7)

The nurse encourages a female client with human papillomavirus (HPV) to receive continued follow-up care because she is at risk for: Answers: A. cervical cancer. B. dysmenorrhea. C. infertility. D. dyspareunia.

A. Cervical cancer Clinical studies have confirmed that HPV is the cause of essentially all cases of cervical cancer. Therefore, the client needs continued follow-up for Pap smears. HPV is not associated with an increased risk for infertility, dyspareunia, or dysmenorrhea. (ch. 5)

A nurse is providing care to a woman in labor. The nurse determines that the client is in the active phase based on which assessment findings? Select all that apply. Answers: A. cervical dilation of 6 cm B. contractions every 2 to 3 minutes C. strong desire to push D. cervical effacement of 90% E. contractions every 90 seconds

A. Cervical dilation of 6 cm B. Contractions every 2 to 3 minutes During the active phase, the cervix usually dilates from 4 to 7 cm, with 40% to 80% effacement taking place. Contractions become more frequent (every 2 to 5 minutes) and increase in duration (45 to 60 seconds). Contractions occurring every 90 seconds, a cervical effacement of 90% and a strong desire to push signify the transition phase (ch. 13)

A nurse is conducting a refresher in-service program for a group of neonatal nurses. After teaching the group about hepatic system adaptations after birth, the nurse determines that the teaching was successful when the group identifies which process as reflective of the change of bilirubin from a fat-soluble product to a water-soluble product? Answers: A. conjugation B. jaundice C. hyperbilirubinemia D. hemolysis

A. Conjugation The process in which bilirubin is changed from a fat-soluble product to a water-soluble product is called conjugation. Hemolysis involves the breakdown of blood cells. In the newborn, hemolysis of the red blood cells is the principal source of bilirubin. Jaundice is the manifestation of increased bilirubin in the bloodstream. Hyperbilirubinemia refers to the increased level of bilirubin in the blood. (ch. 17)

A nurse is discussing fetal development with a pregnant woman. The woman is 12 weeks pregnant and asks, "What's happening with my baby?" Which information would the nurse integrate into the response? Select all that apply. Answers: A. continued sexual differentiation B. eyebrows forming C. lanugo present on the head D. startle reflex present E. digestive system becoming active

A. Continued sexual differentiation E. Digestive system becomes active At 12 weeks, sexual differentiation continues and the digestive system shows activity. Eyebrows form and startle reflex is present between weeks 21 and 24. Lanugo on the head appears about weeks 13 to 16. (ch. 10)

A nurse is participating in an online continuing education course that addresses maternal and newborn health. The nurse is engaging in an interactive activity that requires checking off the causes associated with pregnancy-related mortality. Which condition if checked by the nurse as a cause indicates the need to go back and review the information? Answers: A. precipitous labor B. embolism C. hemorrhage D. infection

A. Precipitous labor Most pregnancy-related complications are preventable. The top five leading causes of pregnancy-related mortality are embolism (20%); hemorrhage (17%); preeclampsia and eclampsia (16%); infection (13%); and cardiomyopathy (8%). Obstructed labor, not precipitous labor, is a leading cause. (ch. 1)

A a nurse is conducting a presentation for a group of pregnant women about measures to prevent toxoplasmosis. The nurse determines that additional teaching is needed when the group identifies which measure as preventative? Answers: A. cooking all meat to an internal temperature of 140° F B. washing raw fruits and vegetables before eating them C. avoiding contact with a cat's litter box D. wearing gardening gloves when working in the soil

A. Cooking all meat to an internal temperature of 140 Meats should be cooked to an internal temperature of 160° F. Other measures to prevent toxoplasmosis include peeling or thoroughly washing all raw fruits and vegetables before eating them, wearing gardening gloves when in contact with outdoor soil, and avoiding the emptying or cleaning of a cat's litter box. (ch. 20)

A nurse is explaining to the parents of a child with bladder exstrophy about the care their infant requires. Which measures would the nurse include in the explanation? Select all that apply. Answers: A. covering the area with a sterile, clear, nonadherent dressing B. preparing for surgical intervention in about 2 weeks C. irrigating the surface with sterile saline twice a day D. monitoring drainage through the suprapubic catheter E. administering prescribed antibiotic therapy

A. Covering the area with a sterile, clear, non adherent dressing D. Monitorign drainage through the suprapubic catheter E. Administering prescribed antibiotic therapy Care for an infant with bladder exstrophy includes covering the area with a sterile, clear, nonadherent dressing and irrigating the bladder surface with sterile saline after each diaper change to prevent infection, assisting with insertion and monitoring drainage from suprapubic catheter, administering prescribed antibiotic therapy, and preparing the parents and infant for surgery within 48 hours after birth. (ch. 24)

A client is diagnosed with gestational hypertension and is receiving magnesium sulfate. Which finding would the nurse interpret as indicating a therapeutic level of medication? Answers: A. deep tendons reflexes 2+ B. respiratory rate of 10 breaths/minute C. urinary output of 20 mL per hour D. difficulty in arousing

A. Deep tendon reflexes 2+ With magnesium sulfate, deep tendon reflexes of 2+ would be considered normal and therefore a therapeutic level of the drug. Urinary output of less than 30 mL, a respiratory rate of less than 12 breaths/minute, and a diminished level of consciousness would indicate magnesium toxicity. (ch. 19)

When assessing cervical effacement of a client in labor, the nurse assesses which characteristic? Answers: A. degree of thinning B. extent of opening to its widest diameter C. fetal presenting part D. passage of the mucous plug

A. Degree of thinning Effacement refers to the degree of thinning of the cervix. Cervical dilation refers to the extent of opening at the widest diameter. Passage of the mucous plug occurs with bloody show as a premonitory sign of labor. The fetal presenting part is determined by vaginal examination and is commonly the head (cephalic), pelvis (breech), or shoulder. (ch. 13)

The nurse administers Rho(D) immune globulin to an Rh-negative client after delivery of an Rh-positive newborn based on the understanding that this drug will prevent her from: Answers: A. developing Rh sensitivity. B. becoming Rh positive. C. developing AB antigens in her blood. D. becoming pregnant with an Rh-positive fetus.

A. Developing Rh sensitivity The woman who is Rh-negative and whose infant is Rh-positive should be given Rho(D) immune globulin within 72 hours after childbirth to prevent sensitization. (ch. 16)

A client is diagnosed with pelvic inflammatory disease (PID). When reviewing the client's medical record, what would the nurse expect to find? Select all that apply. Answers: A. discomfort with cervical motion B. lower abdominal tenderness C. oral temperature of 100.4 degrees F D. dysuria E. dysmenorrhea F. multiparity

A. Discomfort with cervical motion B. Lower abdominal tenderness D. Dysuria E. Dsymenorrhea History and physical examination findings of PID include dysmenorrhea, dysuria, lower abdominal tenderness, and cervical motion tenderness. Typically, the client has a fever above 101 degrees F and is nulliparous. (ch. 5)

The nursing student correctly identifies African Americans to have consistently higher infant mortality rates than other ethnic groups. What factors that contribute to this problem can nurses have the greatest impact upon? (Check all that apply.) Answers: A. Discouraging use of tobacco and non-prescribed medications during pregnancy. B. Supporting maternal choices for alternative birthing methods C. Improving maternal educational levels D. Nutritional teaching prior to and during pregnancy E. Encouraging regular maternal prenatal visits

A. Discouraging the use of tobacco and non-prescribed medications during pregnancy D. Nutritional teaching prior to and during pregnancy E. Encouraging regular maternal prenatal visits Causes of increased infant mortality rates include preterm delivery, congenital anomalies and chromosomal defects, SIDS, use of tobacco and medications not approved by the physician. Improved maternal education levels will not prevent infant deaths and support of alternative birthing methods and locations may lead to complications. (ch. 1)

A pregnant woman undergoes a triple/quadruple screen at 16 to 18 weeks' gestation. What would the nurse suspect if the woman's level is decreased? Answers: A. Down syndrome B. cardiac defects C. sickle-cell anemia D. respiratory disorders

A. Down syndrome Decreased levels might indicate Down syndrome or trisomy 18. Sickle cell anemia may be identified by chorionic villus sampling. Levels would be increased with cardiac defects, such as tetralogy of Fallot. It does not detect respiratory disorders. (ch. 10)

The nurse is assessing the skin of a newborn and notes a rash on the newborn's face and chest. The rash consists of small papules and is scattered with no pattern. The nurse interprets this finding as: Answers: A. erythema toxic. B. nevus flames. C. harlequin sign. D. port wine stain.

A. Erythema toxic Erythema toxicum (newborn rash) is a benign, idiopathic, generalized, transient rash that occurs in up to 70% of all newborns during the first week of life. It consists of small papules or pustules on the skin resembling flea bites. The rash is common on the face, chest, and back. One of the chief characteristics of this rash is its lack of pattern. It is caused by the newborn's eosinophils reacting to the environment as the immune system matures. Harlequin sign refers to the dilation of blood vessels on only one side of the body, giving the newborn the appearance of wearing a clown suit. It gives a distinct midline demarcation, which is described as pale on the nondependent side and red on the opposite, dependent side. Nevus flammeus or port wine stain is a capillary angioma located directly below the dermis. It is flat with sharp demarcations and is purple-red. This skin lesion is made up of mature capillaries that are congested and dilated. (ch. 18)

A postpartum woman who has experienced diastasis recti asks the nurse about what to expect related to this condition. Which response by the nurse would be most appropriate? Answers: A. "Exercise will help to improve the muscles." B. "You'll notice that this will fade to silvery lines." C. "You'll notice that your shoe size will increase." D. "Expect the color to lighten somewhat."

A. Exercise will help to improve the muscles Separation of the rectus abdominis muscles, called diastasis recti, is more common in women who have poor abdominal muscle tone before pregnancy. After birth, muscle tone is diminished and the abdominal muscles are soft and flabby. Specific exercises are necessary to help the woman regain muscle tone. Fortunately, diastasis responds well to exercise, and abdominal muscle tone can be improved. Stretch marks (striae gravidarum) fade to silvery lines. The darkened pigmentation of the abdomen (linea nigra), face (melasma), and nipples gradually fades. Parous women will note a permanent increase in shoe size. (ch. 15)

When assessing a family for possible barriers to health care, the mother reports several problems she has been having when coming to her local clinic. The nurse would consider which reported problem as having the greatest impact on this family's health care? Answers: A. family finances B. health care workers' attitudes C. language barrier D. transportation to the clinic

A. Family finances Financial barriers are one of the most important factors that limit care, with the number of children and families either not having any insurance, not having enough insurance to cover services obtained, or not being able to pay for services. Language, health care worker's attitudes, and transportation are also barriers to health care but are not as fundamentally important as finances. (ch. 1)

When describing the structures involved in fetal circulation to a pregnant woman, the nurse describes which structure as the opening between the right and left atrium? Answers: A. foramen ovale B. ductus arteriosus C. ductus venosus D. umbilical artery

A. Foramen ovale The foramen ovale is the opening between the right and left atrium. The ductus venosus connects the umbilical vein to the inferior vena cava. The ductus arteriosus connects the main pulmonary artery to the aorta. The umbilical artery carries blood to the placenta. (ch. 10)

A nurse working as part of a genetics testing team is describing advances in genetics to a group of nurses. After the discussion, the nurse determines that additional information is needed based on which statement made by the group? Answers: A. "Genetic diagnosis is now available as early as the second trimester." B. "Genetic agents may be used in the future to replace drugs." C. "Gene therapy can be used to repair missing genes with normal ones." D. "Genetic testing can identify presymptomatic conditions in children."

A. Genetic diagnosis is now available as early as the second trimester Genetic diagnosis is now possible very early in pregnancy (see Evidence-Based Practice 10.1). Genetic testing can now identify presymptomatic conditions in children and adults. Gene therapy can be used to replace or repair defective or missing genes with normal ones. Gene therapy has been used for a variety of disorders, including cystic fibrosis, melanoma, diabetes, HIV, and hepatitis (Tamura, Kamuma, Nakazato, et al., 2010). The potential exists for creation of increased intelligence and size through genetic intervention. Recent research using gene therapy shows promise for the generation of insulin-producing cells to cure diabetes (Calne, Gan, & Lee, 2010). In the future, genetic agents may replace drugs, general surgery may be replaced by gene surgery, and genetic intervention may replace radiation. (ch. 10)

A client is admitted to the labor and birthing suite in early labor. On review of her prenatal history, the nurse determines that the client's pelvic shape as identified in the antepartal progress notes is the most favorable one for a vaginal delivery. Which pelvic shape would the nurse have noted? Answers: A. gynecoid B. android C. platypelloid D. anthropoid

A. Gynecoid The most favorable pelvic shape for vaginal delivery is the gynecoid shape. The anthropoid pelvis is favorable for vaginal birth, but it is not the most favorable shape. The android pelvis is not considered favorable for a vaginal birth because descent of the fetal head is slow and failure of the fetus to rotate is common. Women with a platypelloid pelvis usually require cesarean birth. (ch. 13)

A nurse is reviewing a journal article about vaccines used to prevent STIs. The nurse would expect to find information about vaccines for which STIs? Select all that apply. Answers: A. HAV B. HIV C. HBV D. HPV E. HSV

A. HAV C. HBV d. HPV Vaccines are under development or are undergoing clinical trials for certain STIs, including HIV and HSV. However, the only vaccines currently available are for prevention of HAV, HBV, and HPV infection. (ch. 5)

A nurse is auscultating the chest of a client at 16 weeks' gestation. The nurse immediately notifies the health care provider about which finding? Answers: A. heart rate 25 bpm above baseline B. symmetrical chest movement. C. clear breath sounds D. soft systolic murmur

A. Heart rate 25bpm above baseline Heart rate typically increases by 10 to 15 bpm starting between 14 to 20 weeks of pregnancy. However, an increase of 25 bpm would be a cause for concern. A soft systolic murmur, clear breath sounds, and symmetrical chest movement are normal findings. (ch. 12)

As part of discharge planning, the nurse refers a woman to Reach to Recovery. The nurse initiates this referral to facilitate which goal? Answers: A. help support women who have undergone mastectomies B. provide all supplies needed after breast surgery for no cost C. collect statistics for research for the American Cancer Society D. raise funds to support early breast cancer detection programs

A. Help support women who have undergone mastectomies Reach for Recovery is an organization that gives women and their families opportunities to express their feelings, verbalize their fears, and get answers. Reach to Recovery volunteers provide living proof that people can survive breast cancer and lead productive lives. Reach to Recovery helps raise funds, provide supplies, and collect statistics, but these are not the program's primary purpose. (ch. 6)

The nurse is reviewing the laboratory test results of a client with dysfunctional uterine bleeding (DUB). Which finding would be of concern? Answers: A. hemoglobin level of 10.1 g/dL B. negative pregnancy test C. serum cholesterol of 140 mg/dL D. prothrombin time of 60 seconds

A. Hemoglobin level of 10.1 A hemoglobin level of 10.1 g/dL suggests anemia, which might occur secondary to prolonged or heavy menses. A negative pregnancy test, a prothrombin time of 60 seconds, and a serum cholesterol level of 140 mg/dL are within normal parameters. (ch. 4)

A client is suspected of having a ruptured ectopic pregnancy. Which assessment would the nurse identify as the priority? Answers: A. hemorrhage B. jaundice C. edema D. infection

A. Hemorrhage With a ruptured ectopic pregnancy, the woman is at high risk for hemorrhage. Jaundice, edema, and infection are not associated with a ruptured ectopic pregnancy. (ch. 19)

When the nurse is assessing a pregnant woman in her last trimester, which question would be most appropriate to use to gather information about weight gain and fluid retention? Answers: A. "How swollen do your ankles appear before you go to bed? B. "What's your usual dietary intake for a typical day?" C. "How puffy does your face look by the end of a day?" D. "What size maternity clothes are you wearing now?"

A. How swollen do your ankles appear before you go to bed? Edema, especially in the dependent areas such as the legs and feet, occurs throughout the day due to gravity. It improves after a night's sleep. Therefore, questioning the client about ankle swelling would provide the most valuable information. Asking about her usual dietary intake would be valuable in assessing complaints of heartburn and indigestion. The size of maternity clothing may provide information about weight gain but would have little significance for fluid retention. Swelling in the face may suggest preeclampsia, especially if it is accompanied by dizziness, blurred vision, headaches, upper quadrant pain, or nausea. (ch. 12)

After teaching a pregnant woman about the hormones produced by the placenta, the nurse determines that the teaching was successful when the woman identifies which hormone produced as being the basis for pregnancy tests? Answers: A. human chorionic gonadotropin (hCG) B. estrogen (estriol) C. progesterone (progestin) D. human placental lactogen (hPL)

A. Human chorionic gonadotropin (hCG) The placenta produces hCG, which is the basis for pregnancy tests. This hormone preserves the corpus luteum and its progesterone production so that the endometrial lining is maintained. Human placental lactogenmodulates fetal and maternal metabolism and participates in the development of the breasts for lactation. Estrogen causes enlargement of the woman's breasts, uterus, and external genitalia and stimulates myometrial contractility. Progesterone maintains the endometrium. (ch. 10)

A woman comes to the clinic for a routine check-up. After obtaining the client's history, the nurse identifies that the client is at increased risk for cervical cancer based on her history of exposure to which virus? Answers: A. human papillomavirus B. Epstein-Barr virus C. cytomegalovirus D. hepatitis

A. Human-papillomavirus Human papillomavirus is a major causative factor for cervical cancer. Hepatitis, cytomegalovirus, and Epstein-Barr virus are not associated with the development of cervical cancer. (ch. 8)

A couple comes to the clinic for a fertility evaluation. The male partner is to undergo a semen analysis. After teaching the partner about this test, which client statement indicates that the client has understood the instructions? Answers: A. "I have to abstain from sexual activity for about 1 to 2 days before the sample." B. "I will withdraw before I ejaculate during sex to collect the specimen." C. "I need to bring the specimen to the lab the day after collecting it." D. "I will place the specimen in a special plastic bag to transport it."

A. I have to abstain from sexual activity for about 1 to 2 days before the sample Semen analysis is the most important indicator of male fertility. The man should abstain from sexual activity for 24 to 48 hours before giving the sample. For a semen examination, the man is asked to produce a specimen by ejaculating into a specimen container and delivering it to the laboratory for analysis within 1 to 2 hours. When the specimen is brought to the laboratory, it is analyzed for volume, viscosity, number of sperm, sperm viability, motility, and sperm shape. (ch. 4)

A nurse is reviewing the medical record of a pregnant woman and notes that she is gravid II. The nurse interprets this to indicate the number of: Answers: A. pregnancies. B. spontaneous abortions. C. preterm births. D. births.

A. Pregnancies Gravida refers to a pregnant woman—gravida I (primigravida) during the first pregnancy, gravida II (secundigravida) during the second pregnancy, and so on. Para refers to the number of births at 20 weeks or greater that a woman has, regardless of whether the newborn is born alive or dead. "A" would be used to denote the number of abortions and "P" would be used to denote the number of preterm births when using the GTPAL system. (ch. 12)

After the nurse teaches a client about ways to reduce the symptoms of premenstrual syndrome, which client statement indicates a need for additional teaching? Answers: A. "I will make sure to take my estrogen supplements a week before my period." B. "I quit smoking about a month ago, so that should help." C. "I'll cut down on the amount of coffee and colas I drink." D. "I've signed up for an aerobic exercise class three times a week."

A. I will make sure to take my estrogen supplements a week before my period Lifestyle changes such as exercising, avoiding caffeine, and smoking cessation are a key component for managing the signs and symptoms of premenstrual syndrome. Estrogen supplements are not used. If medication is necessary, NSAIDs may be used for painful physical symptoms; spironolactone may help with bloating and water retention. (ch. 4)

A nurse is teaching a woman with mild preeclampsia about important areas that she needs to monitor. The nurse determines that the teaching was successful based on which statement by the woman? Select all that apply. Answers: A. "I'll call my health care provider if I have burning when I urinate." B. "I should complete a fetal kick count each day." C. "I will weigh myself once a week." D. "I will check my urine for protein four times a day." E. "I should check my blood pressure twice a day."

A. I'll call my health care provider if I have burning when I urinate B. I should complete a fetal kick count each day E. I should check my blood pressure twice a day The client should take her blood pressure twice daily, check and record weight daily, perform urine dipstick checks for protein twice daily, record the number of fetal kicks daily, and notify her health care provider is she experiences burning on urination. (ch. 19)

A nurse is preparing a presentation for a local community group about health status and children's health. Which rate would the nurse include as one of the most significant measures? Answers: A. infant mortality rate B. maternal mortality rate C. fetal mortality rate D. neonatal mortality rate

A. Infant mortality rate The infant mortality rate is used as an index of the general health of a country. Generally, this statistic is one of the most significant measures of children's health. (ch. 1)

A nurse is assessing a newborn who is about 41/2 hours old. The nurse would expect this newborn to exhibit which behavior? Select all that apply. Answers: A. interest in environmental stimuli B. spontaneous Moro reflexes C. passage of meconium D. sleeping E. difficulty arousing the newborn

A. Interest in environmental stimuli C. Passage of meconium The newborn is in the second period of reactivity, which begins as the newborn awakens and shows an interest in environmental stimuli. This period lasts 2 to 8 hours in the normal newborn (Boxwell, 2010). Heart and respiratory rates increase. Peristalsis also increases. Thus, it is not uncommon for the newborn to pass meconium or void during this period. In addition, motor activity and muscle tone increase in conjunction with an increase in muscular coordination. Spontaneous Moro reflexes are noted during the first period of reactivity. Sleeping and difficulty arousing the newborn reflect the period of decreased responsiveness. (ch. 17)

A woman in the 34th week of pregnancy says to the nurse, "I still feel like having intercourse with my husband." The woman's pregnancy has been uneventful. The nurse responds based on the understanding that: Answers: A. it is safe to have intercourse at this time. B. intercourse at this time is likely to result in premature labor. C. there are other ways that the couple can satisfy their needs. D. intercourse at this time is likely to cause rupture of membranes.

A. It is safe to have intercourse at this time Sexual activity is permissible during pregnancy unless there is a history of vaginal bleeding, placenta previa, risk of preterm labor, multiple gestation, incompetent cervix, premature rupture of membranes, or presence of infection. Rupture of membranes or premature labor is unlikely since the woman's pregnancy has been uneventful so far. Alternative sexual positions may be necessary as the woman's abdomen increases in size. (ch. 12)

A nurse is conducting a class for a group of teenage girls about female reproductive anatomy and physiology. Which structures would the nurse include as an external female reproductive organ? Select all that apply. Answers: A. labia B. uterus C. clitoris D. vagina E. mons pubis

A. Labia C. Clitoris E. Mons pubis The external female reproductive organs collectively are called the vulva (which means "covering" in Latin). The vulva serves to protect the urethral and vaginal openings and is highly sensitive to touch to increase the female's pleasure during sexual arousal (Stables & Rankin, 2010). The structures that make up the vulva include the mons pubis, the labia majora and minora, the clitoris, the structures within the vestibule, and the perineum. The vagina and uterus are internal female reproductive organs. (ch. 3)

When preparing a woman with suspected vulvar cancer for a biopsy, the nurse expects that the lesion would most likely be located at which area? Answers: A. labia majora B. clitoris C. labia minora D. prepuce

A. Labia majora The diagnosis of vulvar cancer is made by biopsy of the suspicious lesion, which is most commonly found on the labia majora. (ch. 8)

A pregnant client comes to the emergency department experiencing preterm labor with ruptured membranes. After obtaining the client's history, the nurse would identify which factor as most likely contributing to the possibility of a poor outcome for this client? Answers: A. lack of prenatal care B. lack of employment C. lack of adequate nutrition D. lack of formal education

A. Lack of prenatal care Lack of prenatal care during pregnancy is a major factor that contributes to the poor outcome. Prenatal care is well known to prevent complications of pregnancy and to support the birth of healthy infants. (ch. 1)

A nurse is teaching a group of pregnant women about the adverse effects of substances on the fetus. The nurse determines that additional teaching is needed when the group identifies which substance as being teratogenic? Answers: A. marijuana B. nicotine C. cocaine D. alcohol

A. Marijuana Marijuana has not been shown to have teratogenic effects on the fetus. Alcohol, nicotine, and cocaine do affect the fetus. (ch. 24)

When reviewing the medical record of a client diagnosed with endometriosis, the nurse would identify which factor as a risk for this woman? Answers: A. menstrual cycles of 24 days B. age of 14 years for menarche C. short menstrual flow D. low fat in the diet

A. Menstrual cycle of 24 days Risk factors for developing endometriosis include increasing age, family history of endometriosis in a first-degree relative, short menstrual cycle (less than 28 days), long menstrual flow (more than 1 week), high dietary fat consumption, young age at menarche (younger than age 12), and few (one or two) or no pregnancies. (ch. 4)

A woman in her second trimester comes for a follow-up visit and says to the nurse, "I feel like I'm on an emotional roller-coaster." Which response by the nurse would be most appropriate? Answers: A. "Mood swings are completely normal during pregnancy." B. "How often has this been happening to you?" C. "Have you been experiencing any thoughts of harming yourself?" D. "Maybe you need some medication to level things out."

A. Mood swings are completely normal during pregnancy Emotional lability is characteristic throughout most pregnancies. One moment a woman can feel great joy, and within a short time she can feel shock and disbelief. Frequently, pregnant women will start to cry without any apparent cause. Some women feel as though they are riding an "emotional roller-coaster." These extremes in emotion can make it difficult for partners and family members to communicate with the pregnant woman without placing blame on themselves for their mood changes. Clear explanations about how common mood swings are during pregnancy are essential. (ch. 11)

A nurse is observing a postpartum client interacting with her newborn and notes that the mother is engaging with the newborn in the en face position. Which behavior would the nurse be observing? Answers: A. mother making eye-to-eye contact with the newborn B. mother gently stroking the newborn's face C. mother holding the newborn upright at the shoulder D. mother placing the newborn next to bare breast

A. Mother making eye-to-eye contact with the newborn The en face position is characterized by the mother interacting with the newborn through eye-to-eye contact while holding the newborn. (ch. 16)

A nurse is assessing a pregnant woman on a routine checkup. When assessing the woman's gastrointestinal tract, what would the nurse expect to find? Select all that apply. Answers: A. nausea B. hyperemic gums C. heartburn D. complaints of bloating E. increased peristalsis

A. Nausea B. Hyperemic gums C. Heartburn D. Complaints of bloating Gastrointestinal system changes include hyperemic gums due to estrogen and increased proliferation of blood vessels and circulation to the mouth; slowed peristalsis; acid indigestion and heartburn; bloating and nausea and vomiting. (ch. 11)

A client expresses concern that her 2-hour-old newborn is sleepy and difficult to awaken. The nurse explains that this behavior indicates: Answers: A. normal progression of behavior. B. physiological abnormality. C. probable hypoglycemia. D. inadequate oxygenation.

A. Normal progression of behavior From 30 to 120 minutes of age, the newborn enters the second stage of transition, that of sleep or a decrease in activity. More information would be needed to determine if hypoglycemia, a physiologic abnormality, or inadequate oxygenation was present. (ch. 17)

A nurse is working as part of a committee to establish policies to promote bonding and attachment. Which practice would be least effective in achieving this goal? Answers: A. offering round-the-clock nursery care for all infants B. allowing unlimited visiting hours on maternity units C. encouraging infant contact immediately after birth D. promoting rooming-in

A. Offering round-the-clock nursery care for all infants Factors that can affect attachment include separation of the infant and parents for long times during the day, such as if the infant was being cared for in the nursery throughout the day. Unlimited visiting hours, rooming-in, and infant contact immediately after birth promote bonding and attachment. (ch. 16)

When assessing several women for possible VBAC, which woman would the nurse identify as being the best candidate? Answers: A. one who had a previous cesarean birth via a low transverse incision B. one who has a history of a contracted pelvis C. one who has a vertical incision from a previous cesarean birth D. one who has undergone a previous myomectomy

A. One who had a previous cesarean birth via a low transverse incision VBAC is an appropriate choice for women who have had a previous cesarean birth with a lower abdominal transverse incision. It is contraindicated in women who have a prior classic uterine incision (vertical), prior transfundal surgery, such as myomectomy, or a contracted pelvis. (ch. 21)

A nurse is assessing a client diagnosed with pelvic inflammatory disease (PID). Which findings would the nurse most likely assess? Select all that apply. Answers: A. painful urination B. right upper quadrant pain C. tenderness with cervical motion D. oral temperature of 102 degrees F E. Clear vaginal discharge F. negative pregnancy history

A. Painful urination C. Tenderness with cervical motion D. Oral temperature of 102 degrees F F. Negative pregnancy history History and physical examination findings of PID include dysmenorrhea, dysuria, lower abdominal tenderness, cervical or vaginal mucopurulent discharge, and cervical motion tenderness. Typically, the client has a fever above 101 degrees F and is nulliparous.

A nurse is working as part of group to develop programs that address factors affecting maternal, newborn, and women's health. The group demonstrates understanding of the information when they target programs to address which deficiencies? Select all that apply. Answers: A. political representation B. mobility C. literacy D. employment opportunities E. skills

A. Political representation B. Mobility C. Literacy D. Employment opportunities E. Skills Poverty, particularly for women, is more than monetary deficiency. Women continue to lag behind men in control of cash, credit, and collateral. Other forms of impoverishment may include deficiencies in literacy, education, skills, employment opportunities, mobility, and political representation, as well as pressures on time and energy linked to their responsibilities. (ch. 1)

In a client's seventh month of pregnancy, she reports feeling "dizzy, like I'm going to pass out, when I lie down flat on my back." The nurse explains that this is due to: Answers: A. pressure of the gravid uterus on the vena cava. B. pressure of the presenting fetal part on the diaphragm. C. a 50% increase in blood volume. D. physiologic anemia due to hemoglobin decrease.

A. Pressure of the gravid uterus on the vena cava The client is describing symptoms of supine hypotension syndrome, which occurs when the heavy gravid uterus falls back against the superior vena cava in the supine position. The vena cava is compressed, reducing venous return, cardiac output, and blood pressure, with increased orthostasis. The increased blood volume and physiologic anemia are unrelated to the client's symptoms. Pressure on the diaphragm would lead to dyspnea. (ch. 11)

A nurse is preparing a class for a group of women at a family planning clinic about contraceptives. When describing the health benefits of oral contraceptives, which benefits would the nurse most likely include? Select all that apply. Answers: A. reduced risk for endometrial cancer B. reduced risk for migraine headaches C. decreased risk for depression D. improvement in acne E. protection against pelvic inflammatory disease

A. Reduced risk for endometrial cancer D. Improvement in acne E. Protection against pelvic inflammatory disease The health benefits of oral contraceptives include protection against pelvic inflammatory disease, a reduced risk for endometrial cancer, and improvement in acne. Oral contraceptives are associated with an increased risk for depression and migraine headaches. (ch. 4)

A nurse is providing care to a pregnant client during a prenatal visit to the clinic. The woman is in her first trimester, and the nurse is describing the various hormones being produced. Which hormone would the nurse describe as being produced by the placenta? Select all that apply. Answers: A. relaxin B. progestin C. estriol D. prolactin E. human chorionic somatomammotropin

A. Relaxin B. Progestin C. Estriol E. Human chorionic somatomammotropin Estriol, relaxin, progestin, and human chorionic somatomammotropin are secreted by the placenta. Prolactin is secreted after delivery for breast-feeding. (ch. 10)

A nurse is assessing a woman in labor. Which finding would the nurse identify as a cause for concern during a contraction? Answers: A. respiratory rate of 10 breaths/minute B. heart rate increase from 76 bpm to 90 bpm C. blood pressure rise from 110/60 mm Hg to 120/74 D. white blood cell count of 12,000 cells/mm3

A. Respiratory rate of 10 bpm During labor, the mother experiences various physiologic responses including an increase in heart rate by 10 to 20 bpm, a rise in blood pressure by up to 35 mm Hg during a contraction, an increase in white blood cell count to 25,000 to 30,000 cells/mm3, perhaps as a result of tissue trauma, and an increase in respiratory rate with greater oxygen consumption due to the increase in metabolism. A drop in respiratory rate would be a cause for concern. (ch. 13)

The nurse notifies the obstetrical team immediately because the nurse suspects that the pregnant woman may be exhibiting signs and symptoms of amniotic fluid embolism. Which findings would the nurse most likely assess? Select all that apply. Answers: A. significant difficulty breathing B. pulmonary edema C. tachycardia D. hypertension E. bleeding with bruising

A. Significant difficulty breathing B. Pulmonary edema C. Tachycardia E. Bleeding with bruising The clinical appearance is varied, but most women report difficulty breathing. Other symptoms include hypotension, cyanosis, seizures, tachycardia, coagulation failure, disseminated intravascular coagulation, pulmonary edema, uterine atony with subsequent hemorrhage, adult respiratory distress syndrome, and cardiac arrest. (ch. 21)

A woman comes to the prenatal clinic suspecting that she is pregnant, and assessment reveals probable signs of pregnancy. Which finding would the nurse most likely assess? Select all that apply. Answers: A. softening of the cervix B. auscultation of a fetal heart beat C. positive pregnancy test D. absence of menstruation E. ultrasound visualization of the fetus F. ballottement

A. Softening of the cervix C. Positive pregnancy test F. Ballottement Probable signs of pregnancy include a positive pregnancy test, ballottement, and softening of the cervix (Goodell's sign). Ultrasound visualization of the fetus, auscultation of a fetal heart beat, and palpation of fetal movements are considered positive signs of pregnancy. Absence of menstruation is a presumptive sign of pregnancy. (ch. 11)

The nurse is teaching a couple about the pros and cons of genetic testing. Which statement by the nurse best describes the limits of genetic testing? Answers: A. "Some genetic tests can give a probability for developing a disorder." B. "Genetic tests identify people at high risk for preventable conditions." C. "Various genetic tests help the physician choose appropriate treatments." D. "Genetic testing helps couples avoid having children with fatal diseases."

A. Some genetic tests can give a probability for developing a disorder The fact that some tests only provide a probability for developing a disorder raises a problem. A serious limitation of these susceptibility tests is that some people who carry a disease-associated mutation never develop the disease. Choosing appropriate treatments, avoiding having children with fatal diseases, and identifying those at high risk affirm the value of genetic tests. (ch. 10)

A nurse is describing how the fetus moves through the birth canal. Which component would the nurse identify as being most important in allowing the fetal head to move through the pelvis? Answers: A. sutures B. fontanelles C. frontal bones D. biparietal diameter

A. Sutures Sutures are important because they allow the cranial bones to overlap in order for the head to adjust in shape (elongate) when pressure is exerted on it by uterine contractions or the maternal bony pelvis. Fontanelles are the intersections formed by the sutures. The frontal bones, along with the parietal and occipital bones are bones of the cranium that are soft and pliable. The biparietal diameter is an important diameter that can affect the birth process. (ch. 13)

The nurse is teaching a health education class on male reproductive anatomy and asks the students to identify the site of sperm production. Which structure, if identified by the group, would indicate to the nurse that the teaching was successful? Answers: A. testes B. scrotum C. seminal vesicles D. prostate gland

A. Testes The testes are responsible for sperm production. The seminal vesicles produce nutrient seminal fluid. The scrotum surrounds and protects the testes. The prostate gland and the seminal vesicles produce fluid to nourish the sperm. (ch. 3)

A woman in her 40th week of pregnancy calls the nurse at the clinic and says she's not sure whether she is in true or false labor. Which statement by the client would lead the nurse to suspect that the woman is experiencing false labor? Answers: A. "The contractions slow down when I walk around." B. "I'm feeling contractions mostly in my back." C. "My contractions are about 6 minutes apart and regular." D. "If I try to talk to my partner during a contraction, I can't."

A. The contractions slow down when I walk around False labor is characterized by contractions that are irregular and weak, often slowing down with walking or a position change. True labor contractions begin in the back and radiate around toward the front of the abdomen. They are regular and become stronger over time; the woman may find it extremely difficult if not impossible to have a conversation during a contraction (ch. 13)

A client comes to the prenatal clinic for her first visit. When determining the client's estimated due date, the nurse understands what which method is the most accurate? Answers: A. ultrasound B. gestational wheel C. birth calculator D. Nagele's rule

A. Ultrasound Although there are several methods for determining the EDD, the ultrasound is considered the most accurate method for dating the pregnancy (ch. 12)

A client comes to the clinic for an evaluation. After assessing the client, the nurse suspects that the client may be experiencing uterine prolapse. Which finding would the nurse most likely report when notifying the primary care provider about the suspicion? Select all that apply. Answers: A. urge to defecate B. nocturnal urinary frequency C. dyspareunia D. low back pain on sitting E. abdominal pressure

A. Urge to defecate B. Nocturnal urinary frequency D. Abdominal pressure Symptoms associated with pelvic organ prolapse including urgency of defecation, diurnal and nocturnal frequency, abdominal pressure and pain, low back pain on standing for long periods and dyspareunia. (ch. 7)

A nurse is reviewing an article about preterm premature rupture of membranes. Which factors would the nurse expect to find placing a woman at high risk for this condition? Select all that apply. Answers: A. urinary tract infection B. low socioeconomic status C. single gestations D. high body mass index E. smoking

A. Urinary tract infection B. Low socioeconomic status E. Smoking High-risk factors associated with preterm PROM include low socioeconomic status, multiple gestation, low body mass index, tobacco use, preterm labor history, placenta previa, abruptio placenta, urinary tract infection, vaginal bleeding at any time in pregnancy, cerclage, and amniocentesis. (ch. 19)

A nurse is describing the various birth methods to pregnant couples. Which information would the nurse include as part of the Lamaze method? Answers: A. use of specific breathing and relaxation techniques B. concentration on sensations while turning on to own bodies C. interruption of the fear-tension-pain cycle D. focus on the pleasurable sensations of childbirth

A. Use of specific breathing and relaxation techniques Lamaze is a psychoprophylactic ("mind prevention") method of preparing for labor and birth that promotes the use of specific breathing and relaxation techniques. The Bradley method emphasizes the pleasurable sensations of childbirth, teaching women to concentrate on these sensations while "turning on" to their own bodies. The Dick-Read method seeks to interrupt the circular pattern of fear, tension, and pain during the labor and birthing process. (ch. 12)

A 24-year-old client who is planning to become pregnant comes to the clinic for an evaluation. When assessing the client, which finding would alert the nurse to implement measures to reduce the client's risk for problems during pregnancy? Select all that apply. Answers: A. uses ibuprofen daily B. follows a vegetarian diet C. quit smoking 4 years ago D. has a BMI of 22 E. drinks wine 3 to 4 times/week

A. Uses ibuprofen daily E. Drinks wine 3-4 times/wk The use of alcohol and prescription and over-the-counter drugs can be harmful to a growing fetus. Thus the nurse would need to address these areas with the client. If the client was still smoking, then that too would need to be addressed. Healthy nutrition is important, but being a vegetarian does not necessarily indicate that the client is a nutritional risk. A BMI of 22 is considered normal and would not pose a problem. (ch. 12)

A woman with a history of crack cocaine abuse is admitted to the labor and birth area. While caring for the client, the nurse notes a sudden onset of fetal bradycardia. Inspection of the abdomen reveals an irregular wall contour. The client also reports acute abdominal pain that is continuous. Which condition would the nurse suspect? Answers: A. uterine rupture B. amniotic fluid embolism C. umbilical cord prolapse D. shoulder dystocia

A. Uterine rupture Uterine rupture is associated with crack cocaine use, and generally the first and most reliable sign is sudden fetal distress accompanied by acute abdominal pain, vaginal bleeding, hematuria, irregular wall contour, and loss of station in the fetal presenting part. Amniotic fluid embolism often is manifested with a sudden onset of respiratory distress. Shoulder dystocia is noted when continued fetal descent is obstructed after the fetal head is delivered. Umbilical cord prolapse is noted as the protrusion of the cord alongside or ahead of the presenting part of the fetus. (ch. 21)

A postmenopausal woman with uterine prolapse is being fitted with a pessary. The nurse would be most alert for which side effect? Answers: A. vaginal ulceration B. vaginitis C. increased vaginal discharge D. urinary tract infection

A. Vaginal ulceration Use of a pessary can lead to pressure necrosis. Postmenopausal women with thin vaginal mucosa are highly susceptible to vaginal ulceration. Increased vaginal discharge, urinary tract infections, and vaginitis are possible side effects that could be seen in any woman fitted with a pessary. (ch. 7)

A nurse is reviewing the fetal heart rate pattern and observes abrupt decreases in FHR below the baseline, appearing as a U-shape. The nurse interprets these changes as reflecting which type of deceleration? Answers: A. variable decelerations B. late decelerations C. early decelerations D. prolonged decelerations

A. Variable decelerations Variable decelerations present as visually apparent abrupt decreases in FHR below baseline and have an unpredictable shape on the FHR baseline, possibly demonstrating no consistent relationship to uterine contractions. The shape of variable decelerations may be U, V, or W, or they may not resemble other patterns. Early decelerations are visually apparent, usually symmetrical and characterized by a gradual decrease in the FHR in which the nadir (lowest point) occurs at the peak of the contraction. They are thought to be a result of fetal head compression that results in a reflex vagal response with a resultant slowing of the FHR during uterine contractions. Late decelerations are visually apparent, usually symmetrical, transitory decreases in FHR that occur after the peak of the contraction. The FHR does not return to baseline levels until well after the contraction has ended. Delayed timing of the deceleration occurs, with the nadir of the uterine contraction. Late decelerations are associated with uteroplacental insufficiency. Prolonged decelerations are abrupt FHR declines of at least 15 bpm that last longer than 2 minutes but less than 10 minutes. (ch. 14)

A nurse is teaching the mother of a newborn experiencing cocaine withdrawal about caring for the neonate at home. The mother stopped using cocaine near the end of her pregnancy. The nurse determines that additional teaching is needed when the mother identifies which action as appropriate for her newborn? Answers: A. waking the newborn every hour B. wrapping the newborn snugly in a blanket C. offering a pacifier D. checking the newborn's fontanels

A. Waking the newborn every hour Stimuli need to be decreased. Waking the newborn every hour would most likely be too stimulating. Measures such as swaddling the newborn tightly and offering a pacifier help to decrease irritable behaviors. A pacifier also helps to satisfy the newborn's need for nonnutritive sucking. Checking the fontanels provides evidence of hydration. (ch. 24)

The nurse is discussing the insulin needs of a primaparous client with diabetes who has been using insulin for the past few years. The nurse informs the client that her insulin needs will increase during pregnancy based on the nurse's understanding that the placenta produces: Answers: A. hCG, which increases maternal glucose levels. B. hPL, which deceases the effectiveness of insulin. C. relaxin, which decreases the amount of insulin produced. D. estriol, which interferes with insulin crossing the placenta.

A. hPL, which decreases the effectiveness of insulin hPL acts as an antagonist to insulin, so the mother must produce more insulin to overcome this resistance. If the mother has diabetes, then her insulin need would most likely increase to meet this demand. hCG does not affect insulin and glucose level. Estrogen, not estriol, is believed to oppose insulin. In addition, insulin does not cross the placenta. Relaxin is not associated with insulin resistance. (ch. 11)

The nurse is assessing the respirations of several newborns. The nurse would notify the health care provider for the newborn with which respiratory rate at rest? Answers: A. 46 breaths per minute B. 68 breaths per minute C. 38 breaths per minute D. 54 breaths per minute

B. 69 breaths per minute After respirations are established in the newborn, they are shallow and irregular, ranging from 30 to 60 breaths per minute, with short periods of apnea (less than 15 seconds). Thus a newborn with a respiratory rate below 30 or above 60 breaths per minute would require further evaluation. (ch. 17)

The nurse is reviewing the medical records of several clients. Which client would the nurse expect to have an increased risk for developing osteoporosis? Answers: A. a woman who plays tennis twice a week B. a thin woman with small bones C. a woman who drinks one cup of coffee a day D. a woman of African American descent

B. A thin woman with small bones A woman with a small frame and thin bones is at a higher risk for osteoporosis. Caucasian or Asian women, not African American women, are at higher risk for the condition. A woman who plays tennis twice a week is active and thus would be at low risk for osteoporosis. Women who ingest excessive amounts of caffeine are at increased risk. (ch. 4)

A nurse suspects that a pregnant client may be experiencing abruptio placenta based on assessment of which finding? Select all that apply. Answers: A. absence of pain B. absent fetal heart tones C. dark red vaginal bleeding D. rigid uterus E. insidious onset

B. Absent fetal heart tones C. Dark red vaginal bleeding D. Rigid uterus E. Insidious onset Assessment findings associated with abruptio placenta include a sudden onset, with concealed or visible bleeding, dark red bleeding, constant pain or uterine tenderness on palpation, firm to rigid uterine tone, and fetal distress or absent fetal heart tones. (ch. 19)

A client with trichomoniasis is to receive metronidazole. What should the nurse instruct the client to avoid while taking this drug? Answers: A. nicotine B. alcohol C. caffeine D. chocolate

B. Alcohol The client should be instructed to avoid consuming alcohol when taking metronidazole because severe nausea and vomiting could occur. There is no need to avoid nicotine, chocolate, or caffeine when taking metronidazole. (ch. 5)

A nurse is providing family-centered care to a pregnant woman and her family. When implementing care, which action by the nurse would be most appropriate? Answers: A. making the decisions for the woman B. asking the woman about her plans for labor and birth C. focusing on medical procedures associated with birth D. informing the woman what pain relief methods will be used during labor

B. Asking the woman about her plans for labor and birth Family-centered care requires the nurse to apply sensitivity to the client's and family's beliefs and those supporting their culture. This involves listening to the family's needs and shifting the nurse's authoritarian role to the family to empower them to make their own decisions within the context of a supportive environment. The nurse would not make decisions for the woman, focus on the medical aspects of care, or inform the woman about what pain relief methods would be used. Rather the nurse works with the woman and seeks her input about care. These suggestions and advice are incorporated into the client's plan of care. (ch. 1)

The parents of a newborn become concerned when they notice that their baby seems to stop breathing for a few seconds. After confirming the parents' findings by observing the newborn, which action would be most appropriate? Answers: A. Reassure the parents that this is an expected pattern. B. Assess the newborn for signs of respiratory distress. C. Tell the parents not to worry since his color is fine. D. Notify the health care provider immediately.

B. Assess the newborn for signs of respiratory distress Although periods of apnea of less than 20 seconds can occur, the nurse needs to gather additional information about the newborn's respiratory status to determine if this finding is indicative of a developing problem. Therefore, the nurse would need to assess for signs of respiratory distress. Once this information is obtained, then the nurse can notify the health care provider or explain that this finding is an expected one. However, it would be inappropriate to tell the parents not to worry, because additional information is needed. Also, telling them not to worry ignores their feelings and is not therapeutic. (ch. 18)

It is determined that a client's blood Rh is negative and her partner's is positive. To help prevent Rh isoimmunization, the nurse would expect to administer Rho(D) immune globulin at which time? Answers: A. at 32 weeks' gestation and immediately before discharge B. at 28 weeks' gestation and again within 72 hours after delivery C. in the first trimester and within 2 hours of delivery D. 24 hours before delivery and 24 hours after delivery

B. At 28 weeks' gestation and again within 72 hours after delivery To prevent isoimmunization, the woman should receive Rho(D) immune globulin at 28 weeks and again within 72 hours after delivery. (ch. 19)

A primipara client gave birth vaginally to a healthy newborn girl 12 hours ago. The nurse palpates the client's fundus. Which finding would the nurse identify as expected? Answers: A. four fingerbreadths below the umbilicus B. at the level of the umbilicus C. two fingerbreadths above the umbilicus D. two fingerbreadths below the umbilicus

B. At the level of the umbilicus During the first few days after birth, the uterus typically descends downward from the level of the umbilicus at a rate of 1 cm (1 fingerbreadth) per day so that by 3 days, the fundus lies 2 to 3 fingerbreadths below the umbilicus (ch. 15)

A nurse is providing care to a woman in labor. After assessment of the fetus, the nurse documents the fetal lie. Which term would the nurse most likely use? Answers: A. longitudinal B. cephalic C. extension D. flexion

B. Longitudinal Fetal lie refers to the relationships of the long axis (spine) of the fetus to the long axis (spine) of the mother. There are two primary lies: longitudinal and transverse. Flexion and extension are terms used to describe fetal attitude. Cephalic is a term used to describe fetal presentation. (ch. 13)

A nursing group is examining their hospital's maternal outcomes for the previous 5 years. Which identified factors have contributed to the decline in the maternal mortality rate? Select all that apply. Answers: A. Increased use of anesthesia with childbirth B. Better management of hemorrhage and infection C. Closer monitoring for complications associated with hypertension of pregnancy D. Use of ultrasound to detect disorders E. Increased participation of women in prenatal care

B. Better management of hemorrhage and infection C. Closer monitoring for complications associated with hypertension in pregnancy D. Use of ultrasound to detect disorders E. Increased participation of women in prenatal care The following factors have contributed to the decline in the maternal mortality rate: increased participation of women in prenatal care, greater detection of disorders such as ectopic pregnancy or placenta previa and prevention of related complications through the use of ultrasound, increased control of complications associated with hypertension of pregnancy, and decreased use of anesthesia with childbirth. (ch. 1)

Which compound would the nurse have readily available for a client who is receiving magnesium sulfate to treat severe preeclampsia? Answers: A. ferrous sulfate B. calcium gluconate C. calcium carbonate D. potassium chloride

B. Calcium gluconate The antidote for magnesium sulfate is calcium gluconate, and this should be readily available in case the woman has signs and symptoms of magnesium toxicity. (ch. 19)

After teaching the parents of a newborn with retinopathy of prematurity (ROP) about the disorder and treatment, which statement by the parents indicates that the teaching was successful? Answers: A. "We can fix the problem with surgery." B. "Can we schedule follow-up vision screenings with the pediatric ophthalmologist now?" C. "We'll make sure to administer eye drops each day for the next few weeks." D. "I'm sure the baby will grow out of it."

B. Can we schedule follow-up vision screening with the pediatric ophthalmologist now? Parents of a newborn with suspected retinopathy of prematurity (ROP) should schedule follow-up vision screenings with a pediatric ophthalmologist every 2 to 3 weeks, depending on the severity of the findings at the initial examination. (ch. 24)

A woman in labor is to receive continuous internal electronic fetal monitoring. The nurse prepares the client for this monitoring based on the understanding that which criterion must be present? Answers: A. a neonatologist to insert the electrode B. cervical dilation of 2 cm or more C. floating presenting fetal part D. intact membranes

B. Cervical dilation of 2 cm or more For continuous internal electronic fetal monitoring, four criteria must be met: ruptured membranes, cervical dilation of at least 2 cm, fetal presenting part low enough to allow placement of the electrode, and a skilled practitioner available to insert the electrode. (ch. 14)

A woman has been in labor for some time, but her membranes have not yet ruptured. Artificial rupture of membranes is being considered. Which assessment findings would support the decision to perform the procedure? Select all that apply. Answers: A. uterine contractions every 2 minutes B. cervical dilation of 4 cm C. cervical effacement 50% D. fetal head at -2 station E. vertex presentation

B. Cervical dilation of 4 cm D. Fetal head at -2 station An amniotomy (artificial rupture of the fetal membranes) may be performed to augment or induce labor when the membranes have not ruptured spontaneously. Doing so allows the fetal head to have more direct contact with the cervix to dilate it. This procedure is performed with the fetal head at -2 station or lower, with the cervix dilated to at least 3 cm. Contractions, effacement, and presentation are not considerations for performing an amniotomy. (ch. 13)

Which finding obtained during a client history would the nurse identify as increasing a client's risk for ovarian cancer? Answers: A. grand multiparity (more than five children) B. consumption of a high-fat diet C. multiple sexual partners D. underweight

B. Consumption of a high-fat diet Risk factors for ovarian cancer include a high-fat diet, obesity, nulliparity, early menarche, late menopause, and increasing age. Having multiple sexual partners is a risk factor for cervical cancer. (ch. 8)

A client states that she is to have a test to measure bone mass to help diagnose osteoporosis. The nurse would most likely plan to prepare the client for: Answers: A. MRI. B. DEXA scan. C. pelvic X-ray. D. ultrasound.

B. DEXA scan The client most likely will be having a DEXA scan, which is a screening test that calculates the mineral content of the bone at the spine and hip. Ultrasound, MRI, and a pelvic X-ray would be of little help in determining bone mass. (ch. 4)

A woman who is 12 hours postpartum had a pulse rate around 80 beats per minute during pregnancy. Now, the nurse finds a pulse of 66 beats per minute. Which of these actions should the nurse take? Answers: A. Obtain an order for a CBC, as it suggests postpartum anemia. B. Document the finding, as it is a normal finding at this time. C. Contact the primary care provider, as it is a first sign of postpartum eclampsia. D. Contact the primary care provider, as it indicates early DIC.

B. Document the finding, as it is a normal finding at this time Pulse rates of 60 to 80 beats per minute (bpm) at rest are normal during the first week after birth. This pulse rate is called puerperal bradycardia. (ch. 16)

A pregnant woman undergoes maternal serum alpha-fetoprotein (MSAFP) testing at 16 to 18 weeks' gestation. What would the nurse suspect if the woman's level is decreased? Answers: A. sickle-cell anemia B. Down syndrome C. cardiac defects D. open neural tube defect

B. Down syndrome Decreased levels might indicate Down syndrome or trisomy 18. Sickle cell anemia may be identified by chorionic villus sampling. MSAFP levels would be increased with cardiac defects, such as tetralogy of Fallot. A triple marker test would be used to determine an open neural tube defect. (ch. 10)

A woman with hyperemesis gravidarum asks the nurse about suggestions to minimize nausea and vomiting. Which suggestion would be most appropriate for the nurse to make? Answers: A. "Lie down for about an hour after you eat." B. "Drink fluids in between meals rather than with meals." C. "Try to eat three large meals a day with less snacking." D. "Make sure that anything around your waist is quite snug."

B. Drink fluids between meals rather than with meals Suggestions to minimize nausea and vomiting include avoiding tight waistbands to minimize pressure on the abdomen, eating small frequent meals throughout the day, separating fluids from solids by consuming fluids in between meals; and avoiding lying down or reclining for at least 2 hours after eating. (ch. 19)

A woman comes to the clinic reporting a cream-colored nipple discharge with the consistency of toothpaste. On examination, the area below the areola is red and slightly swollen, with tortuous tubular swelling. The nurse interprets these findings as suggestive of which disorder? Answers: A. intraductal papilloma B. duct ectasia C. fibroadenoma D. fibrocystic breast disorder

B. Ductus Ectasia Duct ectasia is manifested by nipple discharge, which can be green, brown, straw-colored, red, gray, or cream-colored and the consistency of toothpaste. Subareolar redness and swelling can be noted, along with tortuous tubular swellings beneath the areola. Fibrocystic breast disorder is characterized by lumpy, tender breasts with possible clear to yellow nipple discharge. Intraductal papilloma is manifested by a wart-like growth in the mammary ducts near the nipple that is soft, nontender, mobile, and poorly delineated. A serous, serosanguinous, or watery discharge from the nipple may occur. Fibroadenoma is characterized by a firm, rubbery, well-circumscribed, freely mobile mass, usually located in the upper outer quadrant of the breast. (ch. 6)

A nurse is developing a teaching plan about nutrition for a group of pregnant women. Which recommendations would the nurse include in the discussion? Select all that apply. Answers: A. Keep weight gain to 15 lb. B. Eat three meals with snacking. C. Avoid using diuretics. D. Limit the use of salt in cooking. E. Participate in physical activity.

B. Eat three meals with snacking C. Avoid using diuretics E. Participate in physical activity To promote optimal nutrition, the nurse would recommend gradual and steady weight gain based on the client's prepregnant weight, eating three meals with one or two snacks daily, not restricting the use of salt unless instructed to do so by the health care provider, avoiding the use of diuretics, and participating in reasonable physical activity daily. (ch. 11)

A school nurse who is teaching a health course at the local high school is presenting information on human development and sexuality. When talking about the role of hormones in sexual development, which hormone does the nurse teach the class is the most important for developing and maintaining the female reproductive organs? Answers: A. Androgens B. Estrogen C. Progesterone D. Follicle-stimulating hormone

B. Estrogen Estrogens are responsible for developing and maintaining the female reproductive organs. Progesterone is the most important hormone for conditioning the endometrium in preparation for implantation of the fertilized ovum. Androgens, secreted by the ovaries in small amounts, are involved in the early development of the follicle and affect the female libido. Follicle-stimulating hormone is responsible for stimulating the ovaries to secrete estrogen. (ch. 3)

A school nurse who is teaching a health course at the local high school is presenting information on human development and sexuality. When talking about the role of hormones in sexual development, which hormone does the nurse teach the class is the most important for developing and maintaining the female reproductive organs? Answers: A. Follicle-stimulating hormone B. Estrogen C. Androgens D. Progesterone

B. Estrogen Estrogens are responsible for developing and maintaining the female reproductive organs. Progesterone is the most important hormone for conditioning the endometrium in preparation for implantation of the fertilized ovum. Androgens, secreted by the ovaries in small amounts, are involved in the early development of the follicle and affect the female libido. Follicle-stimulating hormone is responsible for stimulating the ovaries to secrete estrogen. (ch. 3)

After teaching a group of adolescents about female reproductive anatomy, the nurse determines that the teaching was successful when the adolescents identify which structure as the site of fertilization? Answers: A. vagina B. fallopian tubes C. vestibule D. uterus

B. Fallopian tubes Fertilization occurs in the distal portion of the fallopian tubes. The lining of the uterus is shed with menstruation. The vagina connects the external genitalia to the uterus. The vestibule is an oval area enclosed by the labia minora laterally. (ch. 3)

A nurse is conducting an in-service program for a group of nurses working at the women's health facility about the causes of spontaneous abortion. The nurse determines that the teaching was successful when the group identifies which condition as the most common cause of first trimester abortions? Answers: A. uterine fibroids B. fetal genetic abnormalities C. maternal disease D. cervical insufficiency

B. Fetal genetic abnormalities The causes of spontaneous abortion are varied and often unknown. The most common cause for first-trimester abortions is fetal genetic abnormalities, usually unrelated to the mother. Chromosomal abnormalities are more likely causes in first trimester, and maternal disease is more likely in the second trimester. Those occurring during the second trimester are more likely related to maternal conditions, such as cervical insufficiency, congenital, or acquired anomaly of the uterine cavity (uterine septum or fibroids), hypothyroidism, diabetes mellitus, chronic nephritis, use of crack cocaine, inherited and acquired thrombophilias, lupus, polycystic ovary syndrome, severe hypertension, and acute infection such as rubella virus, cytomegalovirus, herpes simplex virus, bacterial vaginosis, and toxoplasmosis. (ch. 19)

A newborn was diagnosed with a congenital heart defect and will undergo surgery at a later time. The nurse is teaching the parents about signs and symptoms that need to be reported. The nurse determines that the parents have understood the instructions when they state that they will report which signs? Select all that apply. Answers: A. absence of edema B. fever C. increased respiratory rate D. weight loss E. pale skin

B. Fever C. Increased respiratory rate D. Weight loss Signs and symptoms that need to be reported include weight loss, poor feeding, cyanosis, breathing difficulties, irritability, increased respiratory rate, and fever. (ch. 24)

A client is diagnose with a leiomyoma. The client asks the nurse what this is. The nurse describes this as a: Answers: A. fistula. B. fibroid. C. cyst. D. pelvic organ prolapse.

B. Fibroid Leiomyomas are also called uterine fibroids. Cysts are fluid-filled sac-like structures. A fistula is an abnormal opening. Pelvic organ prolapse is an abnormal descent or herniation of the pelvic organs from their original attachment sites or their normal position in the pelvis. (ch. 7)

The nurse is assessing a preterm newborn who is in the neonatal intensive care unit (NICU) for signs and symptoms of overstimulation. Which sign would the nurse be most likely to assess? Answers: A. increased heart rate B. flaying hands C. eupnea D. increased respirations

B. Flaying hands Overstimulation may have negative effects by reducing oxygenation and causing stress. A newborn reacts to stress by flaying the hands or bringing an arm up to cover the face. When overstimulated, such as by noise, lights, excessive handling, alarms, and procedures, and stressed, heart and respiratory rates decrease and periods of apnea or bradycardia may occur. (ch. 23)

Prenatal testing is used to assess for genetic risks and to identify genetic disorders. In explaining to a couple about an elevated alpha-fetoprotein screening test result, the nurse would discuss the need for: Answers: A. immediate termination of the pregnancy based on results. B. further, more definitive evaluations to conclude anything. C. a more specific determination of the acid-base status. D. special care needed for a Down syndrome infant.

B. Further, more definitive evaluations to conclude anything Increased maternal serum alpha fetoprotein levels may indicate a neural tube defect, Turner syndrome, tetralogy of Fallot, multiple gestation, omphalocele, gastroschisis, or hydrocephaly. Therefore, additional information and more specific determinations need to be done before any conclusion can be made. Down syndrome is associated with decreased maternal serum alpha fetoprotein levels. This type of testing provides no information about the acid-base status of the fetus. Immediate termination is not warranted; more information is needed. (ch. 10)

The nurse would be alert for possible placental abruption during labor when assessment reveals which finding? Answers: A. gestational diabetes B. gestational hypertension C. macrosomia D. low parity

B. Gestational hypertension Risk factors for placental abruption include preeclampsia, gestational hypertension, seizure activity, uterine rupture, trauma, smoking, cocaine use, coagulation defects, previous history of abruption, domestic violence, and placental pathology. Macrosomia, gestational diabetes, and low parity are not considered risk factors. (ch. 21)

A woman gives birth to a healthy newborn. As part of the newborn's care, the nurse instills erythromycin ophthalmic ointment as a preventive measure related to which STI? Answers: A. syphilis B. gonorrhea C. genital herpes D. hepatitis B

B. Gonorrhea To prevent gonococcal ophthalmia neonatorum, erythromycin or tetracycline ophthalmic ointment is instilled into the eyes of all newborns. This action is required by law in most states. The ointment is not used to prevent conditions related to genital herpes, hepatitis B, or syphilis. (ch. 5)

During a vaginal exam, the nurse notes that the lower uterine segment is softened. The nurse documents this finding as: Answers: A. Goodall's sign. B. Hagar's sign. C. Chadwick's sign. D. Ortolans's sign.

B. Hagar's sign Hegar's sign refers to the softening of the lower uterine segment or isthmus. Bluish coloration of the cervix is termed Chadwick's sign. Goodell's sign refers to the softening of the cervix. Ortolani's sign is a maneuver done to identify developmental dysplasia of the hip in infants. (ch. 11)

When describing the neurologic development of a newborn to his parents, the nurse would explain that it occurs in which fashion? Answers: A. lateral-to-medial B. head-to-toe C. outward-to-inward D. distal-caudal

B. Head-to-toe Neurologic development follows a cephalocaudal (head-to-toe) and proximal-distal (center to outside) pattern. (ch. 17)

A pregnant client has received dinoprostone. Following administration of this medication, the nurse assesses the client and determines that the client is experiencing an adverse effect of the medication based on which client report? Select all that apply. Answers: A. hypotension B. headache C. diarrhea D. tachycardia E. nausea

B. Headache C. Diarrhea E. Nausea Adverse effects associated with dinoprostone include headache, nauseas and vomiting, and diarrhea. Tachycardia and hypotension are not associated with this drug. (ch. 21)

A pregnant woman needs an update in her immunizations. Which vaccination would the nurse ensure that the woman receives? Answers: A. rubella B. hepatitis B C. measles D. mumps

B. Hepatitis B Hepatitis B vaccine should be considered during pregnancy. Immunizations for measles, mumps, and rubella are contraindicated during pregnancy. (ch. 12)

When assessing a female client for the possibility of vulvar cancer, the nurse would most likely expect the client to report which symptom? Select all that apply. Answers: A. abnormal Pap smear B. history of herpes simplex C. lesion on the cervix D. abnormal vaginal bleeding E. persistent vulvar itching

B. History of herpes simplex E. Persistent vulvar itching In most cases, the woman with vulvar cancer reports persistent vulvar itching, burning, and edema that does not improve with the use of creams or ointments. A history of condyloma, gonorrhea, and herpes simplex are some of the factors for greater risk for vulvar intraepithelial neoplasia. Abnormal vaginal bleeding, lesion on the cervix, or abnormal Pap smear are not associated with vulvar cancer. (ch. 8)

A nurse is assessing a pregnant woman with gestational hypertension. Which finding would lead the nurse to suspect that the client has developed severe preeclampsia? Answers: A. blood pressure 150/96 mm Hg B. hyperreflexia C. mild facial edema D. urine protein 300 mg/24 hours

B. Hyperreflexia Severe preeclampsia is characterized by blood pressure over 160/110 mm Hg, urine protein levels greater than 500 mg/24 hours, and hyperreflexia. Mild facial edema is associated with mild preeclampsia. (ch. 19)

A client is being discharged after having a right-sided modified radical mastectomy. After teaching the client about ways to minimize lymphedema, the nurse determines that the teaching was successful based on which client statement? Answers: A. "I should have my blood pressure taken in my right arm." B. "I need to avoid wearing tops that have elastic in the sleeves." C. "I need to limit my driving to once a week." D. "I should use lotion on my hands after working in my garden."

B. I need to avoid wearing tops that have elastic in the sleeves Lymphedema increases when there is obstruction to the lymph flow. Wearing clothing with elasticized sleeves would compress the extremity, possibly cause trauma, and obstruct the flow, thus increasing the woman's risk. However, wearing a well-fitted compression sleeve would promote drainage return. Wearing gloves when gardening and using the unaffected arm for blood pressure readings help to reduce the risk of injury and subsequent lymphedema. Driving would have no effect on lymphedema. (ch. 6)

After teaching a woman who has had an evacuation for gestational trophoblastic disease (hydatidiform mole or molar pregnancy) about her condition, which statement indicates that the nurse's teaching was successful? Answers: A. "My intake of iron will have to be closely monitored for 6 months." B. "I will be sure to avoid getting pregnant for at least 1 year." C. "I won't use my birth control pills for at least a year or two." D. "My blood pressure will continue to be increased for about 6 more months."

B. I will be sure to avoid getting pregnant for at least 1 year After evacuation of trophoblastic tissue (hydatiform mole), long-term follow-up is necessary to make sure any remaining trophoblastic tissue does not become malignant. Serial hCG levels are monitored closely for one year, and the client is urged to avoid pregnancy for 1 year because it can interfere with the monitoring of hCG levels. Iron intake and blood pressure are not important aspects of follow-up after evacuation of a hydatiform mole. Use of a reliable contraceptive is strongly recommended so that pregnancy is avoided. (ch. 19)

A nurse is teaching a woman about measures to prevent preterm labor in future pregnancies because the woman just experienced preterm labor with her most recent pregnancy. The nurse determines that the teaching was successful based on which statement by the woman? Answers: A. "Stress isn't a problem that is related to preterm labor." B. "I'll make sure to limit the amount of long distance traveling I do." C. "Separating pregnancies by about a year should be helpful." D. "I'll need extra iron in my diet so I have a extra for the baby."

B. I'll make sure to limit the amount of long distance traveling I do Appropriate measures to reduce the risk for preterm labor include: avoiding travel for long distances in cars, trains, planes or buses; achieving adequate iron store through balanced nutrition (excess iron is not necessary); waiting for at least 18 months between pregnancies, and using stress management techniques for stress. (ch. 21)

The nurse is teaching a couple about X-linked disorders because they are concerned that they might pass on hemophilia to their children. Which response indicates the need for further teaching? Answers: A. "If the mother is a carrier, her sons will have hemophilia." B. "If the father doesn't have it, then his kids won't either." C. "The father can't be a carrier if he doesn't have hemophilia." D. "If the mother is a carrier, her daughter could be one too."

B. If the father doesn't have it, then his kids won't either Males are more affected than females. A male has only one X chromosome, and all the genes on his X chromosome will be expressed whereas a female will usually need both X chromosomes to carry the disease. There is no male-to-male transmission (since no X chromosome from the male is transmitted to male offspring), but any man who is affected will have carrier daughters. If a woman is a carrier, there is a 50% chance that her sons will be affected and a 50% chance that her daughters will be carriers. (ch. 10)

A nurse is preparing a presentation for a group of perinatal nurses about common problems associated with preterm birth. When describing the preterm newborn's risk for perinatal asphyxia, the nurse includes which factor as contributing to the newborn's risk? Select all that apply. Answers: A. placental deprivation B. immaturity of the respiratory control centers C. surfactant deficiency D. depleted glycogen stores E. decreased amounts of brown fat

B. Immaturity of the respiratory control centers C. Surfactant deficiency Preterm newborns are at risk for perinatal asphyxia due to surfactant deficiency, unstable chest wall, immaturity of the respiratory control centers, small respiratory passages, and inability to clear mucus from the airways. Placental deprivation places the post-term newborn at risk for perinatal asphyxia. Decreased amounts of brown fat and depleted glycogen stores place the SGA newborn at risk for problems with thermoregulation. (ch. 23)

A nurse is preparing a presentation for a group of new nurses about the potential for misuse of genetic discoveries and advances. Which aspect would the nurse most likely address? Answers: A. slower diagnosis of specific diseases B. individual risk profiling and confidentiality C. greater emphasis on the causes of diseases D. gene replacement therapy for defective genes

B. Individual risk profiling and confidentiality Individual risk profiling based on an individual's genetic makeup can raise issues related to privacy and confidentiality. Gene replacement therapy for defective genes and a greater emphasis on looking at the causes of disease are considered benefits associated with genetic advances. Rapid, more specific diagnosis of diseases would be possible. (ch. 10)

A client experienced prolonged labor with prolonged premature rupture of membranes. The nurse would be alert for which condition in the mother and the newborn? Answers: A. hemorrhage B. infection C. trauma D. hypovolemia

B. Infection Although hemorrhage, trauma, and hypovolemia may be problems, the prolonged labor with the prolonged premature rupture of membranes places the client at high risk for a postpartum infection. The rupture of membranes removes the barrier of amniotic fluid, so bacteria can ascend. (ch. 22)

A client comes to the clinic with abdominal pain. Based on her history the nurse suspects endometriosis. The nurse expects to prepare the client for which evaluatory method to confirm this suspicion? Answers: A. transvaginal ultrasound B. laparoscopy C. pelvic examination D. hysterosalpingogram

B. Laparoscopy The only certain method of diagnosing endometriosis is by seeing it. Therefore, the nurse would expect to prepare the client for a laparoscopy to confirm the diagnosis. A pelvic examination and transvaginal ultrasound are done to assess for endometriosis but do not confirm its presence. Hysterosalpingography aids in identifying tubal problems resulting in infertility. (ch. 4)

A nurse is completing the assessment of a woman admitted to the labor and birth suite. Which information would the nurse expect to include as part of the physical assessment? Select all that apply. Answers: A. estimated date of birth B. membrane status C. contraction pattern D. support system E. fundal height measurement F. current pregnancy history

B. Membrane status C. Contraction pattern E. Fundal height measurement As part of the admission physical assessment, the nurse would assess fundal height, membrane status, and contractions. Current pregnancy history, support systems, and estimated date of birth would be obtained when collecting the maternal health history. (ch. 14)

A woman is diagnosed with a vaginal infection. After teaching the client about measures to reduce her risk, the nurse determines that the client needs additional teaching when she states which factor as increasing her risk? Answers: A. antibiotic therapy B. menstruation C. douching D. use of feminine hygiene sprays

B. Menstruation The vagina has an acidic environment, which protects it against ascending infections. Antibiotic therapy, douching, perineal hygiene sprays, and deodorants upset the acid balance within the vaginal environment and can predispose women to infections. Menstruation is not considered a risk factor. (ch. 3)

A nurse is assessing a woman after birth and notes a second-degree laceration. The nurse interprets this as indicating that the tear extends through which area? Answers: A. anterior rectal wall B. muscles of perineal body C. anal sphincter D. skin

B. Muscles of perineal body The extent of the laceration is defined by depth: a first-degree laceration extends through the skin; a second-degree laceration extends through the muscles of the perineal body; a third-degree laceration continues through the anal sphincter muscle; and a fourth-degree laceration also involves the anterior rectal wall. (ch. 14)

After teaching an in-service program to a group of nurses working in newborn nursery about a neutral thermal environment, the nurse determines that the teaching was successful when the group identifies which process as the newborn's primary method of heat production? Answers: A. bilirubin conjugation B. nonshivering thermogenesis C. cold stress D. convection

B. Nonshivering thermogenesis The newborn's primary method of heat production is through nonshivering thermogenesis, a process in which brown fat (adipose tissue) is oxidized in response to cold exposure. Convection is a mechanism of heat loss. Cold stress results with excessive heat loss that requires the newborn to use compensatory mechanisms to maintain core body temperature. Bilirubin conjugation is a mechanism by which bilirubin in the blood is eliminated. (ch. 17)

A woman just delivered a healthy term newborn. Upon assessing the umbilical cord, the nurse would identify what findings as normal? Select all that apply. Answers: A. one ligament B. one vein C. two veins D. two arteries E. two ligaments F. one artery

B. One vein D. Two arteries The normal umbilical cord contains one large vein and two small arteries. (ch. 10)

A client's maternal serum alpha-fetoprotein (MSAFP) level was unusually elevated at 17 weeks. The nurse suspects which condition? Answers: A. fetal hypoxia B. open spinal defects C. maternal hypertension D. Down syndrome

B. Open spinal defects Elevated MSAFP levels are associated with open neural tube defects. Fetal hypoxia would be noted with fetal heart rate tracings and via nonstress and contraction stress testing. MSAFP in conjunction with marker screening tests would be more reliable for detecting Down syndrome. Maternal hypertension would be noted via serial blood pressure monitoring. (ch. 12)

A preterm newborn has received oxygen therapy during his 3-month stay in the NICU. As the newborn is prepared to be discharged home, the nurse anticipates a referral for which specialist? Answers: A. cardiologist B. ophthalmologist C. neurologist D. nephrologist

B. Ophthalmologist Oxygen therapy has been implicated in the pathogenesis of retinopathy of prematurity (ROP). An ophthalmology consult for follow-up after discharge is essential for preterm infants who have received extensive oxygen. Although referrals to other specialists may be warranted depending on the newborn's status, there is no information to suggest that any would be needed. (ch. 23)

A nurse is conducting an in-service program for a group of nurses working in the labor and birth suite of the facility. After teaching the group about the factors affecting the labor process, the nurse determines that the teaching was successful when the group identifies which component as part of the true pelvis? Select all that apply. Answers: A. vagina B. pelvic inlet C. mid pelvis D. pelvic outlet E. cervix F. pelvic floor muscles

B. Pelvic inlet C. Mid pelvis D. Pelvic outlet The true pelvis is made up of three planes: the pelvic inlet, mid pelvis, and pelvic outlet. The cervix, vagina, and pelvic floor muscles are the soft tissues of the passageway. (ch. 13)

The nurse is preparing an outline for a class on the physiology of the male sexual response. Which event would the nurse identify as occurring first? Answers: A. sperm emission B. penile vasodilation C. psychological release D. ejaculation

B. Penile vasodilation With sexual stimulation, the arteries leading to the penis dilate and increase blood flow into erectile tissue. Blood accumulates, causing the penis to swell and elongate. Sperm emission (movement of sperm from the testes and fluid from the accessory glands) occurs with orgasm. Orgasm results in a pleasurable feeling of physiologic and psychological release. Ejaculation results in the discharge of semen from the urethra. (ch. 3)

The nurse is counseling a couple who suspect that they could bear a child with a genetic abnormality. What would be most important for the nurse to do when working with this family? Answers: A. Maintain the confidentiality of the information. B. Present the information in a nondirective manner. C. Gather information for three generations. D. Inform the family of the need for information.

B. Present the information in a nondriective manner It is essential to respect client autonomy and present information in a factual, nondirective manner. In these situations, the nurse needs to understand that the choice is the couple's to make. Gathering information for three generations obtains a broad overview of what has been seen in both sides of the family. Maintaining confidentiality of the information is as important as with any other client information gathered. Informing the family of the need for information is necessary because of its personal nature. (ch. 10)

After teaching a group of pregnant women about breastfeeding, the nurse determines that the teaching was successful when the group identifies which hormone as important for the production of breast milk after birth? Answers: A. placental estrogen B. prolactin C. progesterone D. gonadotropin-releasing hormone

B. Prolactin After birth and expulsion of the placenta, prolactin stimulates the production of milk. Placental estrogen and progesterone stimulate the development of the mammary glands during pregnancy. Gonadotropin-releasing hormone induces the release of follicle-stimulating hormone and luteinizing hormone to assist with ovulation (ch. 3)

When describing the menstrual cycle to a group of young women, the nurse explains that estrogen levels are highest during which phase of the endometrial cycle? Answers: A. ischemic B. proliferative C. menstrual D. secretory

B. Proliferative Estrogen levels are the highest during the proliferative phase of the endometrial cycle, when the endometrial glands enlarge in response to increasing amounts of estrogen. Progesterone is the predominant hormone of the secretory phase. Levels of estrogen and progesterone drop sharply during the ischemic phase and fall during the menstrual phase. (ch. 3)

A client has an abnormal Pap smear that is classified as ASC-US. Based on the nurse's understanding of this classification, the nurse would expect which procedure? Answers: A. immediate colposcopy B. repeat Pap smear in 4 to 6 months C. cone biopsy D. testing for HPV

B. Repeat Pap smear in 4 to 6 months For the classification of ASC-US, the client would have a repeat Pap smear in 4 to 6 months or be referred for a colposcopy. A referral for colposcopy with HPV testing is indicated if the results indicated ASC-H classification. An immediate colposcopy would be indicated for atypical glandular cells and adenocarcinoma in situ. A cone biopsy would be used to evaluate the lesion and may be used as treatment to remove any precancers and very early cancers. (ch. 8)

A nurse is assessing a newborn. Which finding would alert the nurse to the possibility of respiratory distress in a newborn? Answers: A. symmetrical chest movements B. sternal retractions C. respirations of 40 breaths/minute D. periodic breathing

B. Sternal retractions Sternal retractions, cyanosis, tachypnea, expiratory grunting, and nasal flaring are signs of respiratory distress in a newborn. Symmetrical chest movements and a respiratory rate between 30 to 60 breaths/minute are typical newborn findings. Some newborns may demonstrate periodic breathing (cessation of breathing lasting 5 to 10 seconds without changes in color or heart rate) in the first few days of life. (ch. 17)

A pregnant woman at 31-weeks' gestation calls the clinic and tells the nurse that she is having contractions sporadically. Which instructions would be most appropriate for the nurse to give the woman? Select all that apply. Answers: A. "Walk around the house for the next half hour." B. "Stop what you are doing and rest." C. "Drink 2 or 3 glasses of water." D. "Lie down on your back." E. "Try emptying your bladder."

B. Stop what you are doing and rest C. Drink 2 or 3 glasses of water E. Try emptying your bladder Appropriate instructions for the woman who may be experiencing preterm labor include having the client stop what she is doing and rest for an hour, empty her bladder, lie down on her left side, and drink two to three glasses of water." (ch. 21)

A woman is admitted for repair of cystocele and rectocele. She has nine living children. In taking her health history, what would the nurse expect to find? Answers: A. heavy leukorrhea with vulvar pruritus B. stress incontinence with feeling of low abdominal pressure C. sporadic vaginal bleeding accompanied by chronic pelvic pain D. menstrual irregularities and hirsutism on the chin

B. Stress incontinence with feeling of low abdominal pressure Cystocele and rectocele are examples of pelvic organ prolapse. Manifestations typically include stress incontinence and lower abdominal pressure or pain. Complaints of sporadic vaginal bleeding and chronic pelvic pain are associated with uterine fibroids. Leukorrhea and vulvar pruritus commonly are associated with an infection. Menstrual irregularities and hirsutism are associated with polycystic ovarian syndrome. (ch. 7)

A nurse is assessing a newborn who has been classified as small for gestational age. Which characteristics would the nurse expect to find? Select all that apply. Answers: A. increased amount of breast tissue B. sunken abdomen C. wasted extremity appearance D. adequate muscle tone over buttocks E. narrow skull sutures

B. Sunken abdomen C. Wasted extremity appearance Typical characteristics of SGA newborns include a head that is disproportionately large compared to the rest of the body, wasted appearance of the extremities, reduced subcutaneous fat stores, decreased amount of breast tissue, scaphoid abdomen, wide skull sutures, poor muscle tone over buttocks and cheeks, loose and dry skin appearing oversized, and a thin umbilical cord (ch. 23)

A woman hospitalized with severe preeclampsia is being treated with hydralazine to control blood pressure. Which finding would the lead the nurse to suspect that the client is having an adverse effect associated with this drug? Answers: A. gastrointestinal bleeding B. tachycardia C. sweating D. blurred vision

B. Tachycardia Hydralazine reduces blood pressure but is associated with adverse effects such as palpitation, tachycardia, headache, anorexia, nausea, vomiting, and diarrhea. It does not cause gastrointestinal bleeding, blurred vision, or sweating. Magnesium sulfate may cause sweating. (ch. 19)

A woman comes to the clinic complaining that she has little sexual desire. As part of the client's evaluation, the nurse would anticipate the need to evaluate which hormone level? Answers: A. progesterone B. testosterone C. estrogen D. gonadotropin-releasing hormone

B. Testosterone Testosterone is thought to be the hormone of sexual desire in women. Thus, an evaluation of this level would be done. Progesterone is often called the hormone of pregnancy because of its calming effect (reduction in uterine contractions) on the uterus, allowing pregnancy to be maintained. Estrogen is the predominant hormone at the end of the follicular phase. Gonadotropin-releasing hormone induces the release of FSH and LH to assist with ovulation. (ch. 3)

The nurse is teaching a group of parents about the similarities and differences between newborn skin and adult skin. Which statement by the group indicates that additional teaching is needed? Answers: A. The newborn has fewer fibrils connecting the dermis and epidermis B. The newborn's sweat glands function fully, just like those of an adult. C. Skin development in the newborn is not complete at birth. D. The newborn's skin and that of an adult are similar in thickness.

B. The newborn's sweat glands function fully, just like those of an adult The newborn has sweat glands, like an adult, but full adult functioning is not present until the second or third year of life. The newborn and adult epidermis is similar in thickness and lipid composition, but skin development is not complete at birth. Fewer fibrils connect the dermis and epidermis in the newborn when compared with the adult. (ch. 17)

A nurse is conducting a continuing education program for a group of nurses working in the perinatal unit. After reviewing information about the maternal bony pelvis with the group, the nurse determines that the teaching was successful based on which statement by the group? Answers: A. The bony pelvis plays a lesser role during labor than soft tissue. B. The pelvic outlet is associated with the true pelvis. C. The false pelvis is the passageway through which the fetus travels. D. The false pelvis lies below the imaginary linea terminalis.

B. The pelvic outlet is associated with the true pelvis The maternal bony pelvis consists of the true and false portions. The true pelvis is made up of three planes—the inlet, the mid pelvis, and the outlet. The bony pelvis is more important part of the passageway because it is relatively unyielding. The false pelvis lies above the imaginary linea terminalis. The true pelvis is the bony passageway through which the fetus must travel. (ch. 13)

A nurse is describing the advantages and disadvantages of circumcision to a group of expectant parents. Which statement by the parents indicates effective teaching? Answers: A. "Circumcision is a risk factor for acquiring HIV infection." B. "The rate of penile cancer is less for circumcised males." C. "Urinary tract infections are more easily treated in circumcised males." D. "Sexually transmitted infections are more common in circumcised males."

B. The rate of penile cancer is less for circumcised males The risk for penile cancer appears to be slightly lower for males who are circumcised. However, penile cancer is rare and other risk factors such as genital warts and HPV infection seem to play a larger role. Sexually transmitted infections are less common in circumcised males, but the risk is believed to be related more to behavioral factors than circumcision status. Circumcised males have a 50% lower risk of acquiring HIV infection. Urinary tract infections are slightly less common in circumcised boys. However, rates are low in both circumcised and uncircumcised boys and are easily treated without long-term sequelae. (ch. 18)

A client has not received any medication during her labor. She is having frequent contractions every 1 to 2 minutes and has become irritable with her coach and no longer will allow the nurse to palpate her fundus during contractions. Her cervix is 8 cm dilated and 90% effaced. The nurse interprets these findings as indicating: Answers: A. pelvic phase of the second stage of labor. B. transition phase of the first stage of labor. C. active phase of the first stage of labor. D. latent phase of the first stage of labor.

B. Transition phase of the first stage of labor The transition phase is characterized by cervical dilation of 8 to 10 cm, effacement of 80% to 100%, contractions that are strong, painful, and frequent (every 1 to 2 minutes) and last 60 to 90 seconds, and irritability, apprehension, and feelings of loss of control. The latent phase is characterized by mild contractions every 5 to 10 minutes, cervical dilation of 0 to 3 cm and effacement of 0% to 40%, and excitement and frequent talking by the mother. The active phase is characterized by moderate to strong contractions every 2 to 5 minutes, cervical dilation of 4 to 7 cm and effacement of 40% to 80%, with the mother becoming intense and inwardly focused. The pelvic phase of the second stage of labor is characterized by complete cervical dilation and effacement, with strong contractions every 2 to 3 minutes; the mother focuses on pushing. (ch. 13)

A father of a newborn tells the nurse, "I may not know everything about being a dad, but I'm going to do the best I can for my son." The nurse interprets this as indicating the father is in which stage of adaptation? Answers: A. expectations B. transition to mastery C. reality D. taking-in

B. Transition to mastery The father's statement reflects transition to mastery because he is making a conscious decision to take control and be at the center of the newborn's life regardless of his preparedness. The expectations stage involves preconceptions about how life will be with a newborn. Reality occurs when fathers realize their expectations are not realistic. Taking-in is a phase of maternal adaptation. (ch. 15)

A nurse is teaching a pregnant client in her first trimester about discomforts that she may experience. The nurse determines that the teaching was successful when the woman identifies which discomfort as common during the first trimester? Select all that apply. Answers: A. backache B. urinary frequency C. leg cramps D. cravings E. breast tenderness

B. Urinary frequency D. Cravings E. Breast tenderness Discomforts common in the first trimester include urinary frequency, breast tenderness, and cravings. Backache and leg cramps are common during the second trimester. Legs cramps are also common during the third trimester. (ch. 12)

A woman is scheduled to undergo fetal nuchal translucency testing. Which statement would the nurse include when describing this test? Answers: A. "A small piece of tissue from the fetal part of the placenta is taken." B. "You'll have an intravaginal ultrasound to measure fluid in the fetus." C. "The doctor will take a sample of fluid from your bag of waters." D. "A needle will be inserted directly into the fetus's umbilical vessel."

B. You'll have an intravaginal ultrasound to measure fluid in the fetus Fetal nuchal translucency testing involves an intravaginal ultrasound that measures fluid collection in the subcutaneous space between the skin and cervical spine of the fetus. Insertion of needle into the fetus's umbilical vessel describes percutaneous umbilical blood sampling. Taking a sample of fluid from the amniotic sac (bag of waters) describes an amniocentesis. Obtaining a small tissue specimen from the fetal part of the placenta describes chorionic villus sampling. (ch. 10)

Which statement would the nurse include when teaching a pregnant woman about chorionic villi sampling? Answers: A. "Afterwards, you can resume your exercise program." B. "You'll have an ultrasound first and then the test." C. "This test is very helpful for identifying spinal defects." D. "The results should be available in about a week."

B. You'll have an ultrasound first and then the test With CVS, an ultrasound is done first to localize the embryo. Results are usually available within 48 hours. After the procedure, the woman should refrain from any strenuous activity for the next 48 hours. CVS can be used to detect numerous genetic disorders but not neural tube defects. (ch. 12)

The nurse is providing care to several pregnant women who may be scheduled for labor induction. The nurse identifies the woman with which Bishop score as having the best chance for a successful induction and vaginal birth? Answers: A. 3 B. 8 C. 11 D. 6

C. 11 The Bishop score helps identify women who would be most likely to achieve a successful induction. The duration of labor is inversely correlated with the Bishop score: a score over 8 indicates a successful vaginal birth. Therefore, the woman with a Bishop score of 11 would have the greatest chance for success. Bishop scores of less than 6 usually indicate that a cervical ripening method should be used prior to induction. (ch. 21)

A nurse measures a pregnant woman's fundal height and finds it to be 28 cm. The nurse interprets this to indicate that the client is at how many weeks' gestation? Answers: A. 20 weeks' gestation B. 14 weeks' gestation C. 28 weeks' gestation D. 36 weeks' gestation

C. 28 weeks' gestation Typically, the height of the fundus is measured when the uterus arises out of the pelvis to evaluate fetal growth. At 12 weeks' gestation the fundus can be palpated at the symphysis pubis. At 16 weeks' gestation the fundus is midway between the symphysis and the umbilicus. At 20 weeks the fundus can be palpated at the umbilicus and measures approximately 20 cm from the symphysis pubis. By 36 weeks the fundus is just below the xiphoid process and measures approximately 36 cm. (ch. 12)

A nurse is examining a female client and tests the client's vaginal pH. Which finding would the nurse interpret as normal? Answers: A. 10 B. 7 C. 4.5 D. 8.5

C. 4.5 The vagina has an acidic environment; therefore, a pH of 4.5 would indicate an acidic environment. A pH of 7 is considered neutral; a pH above 7 is considered alkaline. (ch. 3)

A woman comes to the prenatal clinic for an evaluation because she thinks that she may be pregnant. The nurse is assisting the health care provider with the vaginal examination. The exam reveals a vaginal mucosa and cervix that are bluish-purple in color. Based on this information, the nurse suspects that the client is most likely how many weeks pregnant? Answers: A. 16 weeks B. 14 weeks C. 6 weeks D. 5 weeks

C. 6 weeks The finding indicates Chadwick's sign, a bluish-purple discoloration of the vaginal mucosa and cervix. This typically occurs as early as 6 weeks. Goodell's sign (softening of the cervis) occurs at about 5 weeks. Abdominal enlargement typically begins at about 14 weeks and ballottement (when the examiner pushes against the woman's cervix during a pelvic examination and feels a rebound from the floating fetus) usually occurs at about 16 weeks. (ch. 11)

A nurse is providing care to a LGA newborn. The nurse checks the newborn's blood glucose level and finds it to be 23 mg/dL. Which action would the nurse do first? Answers: A. Place the newborn under a radiant warmer. B. Feed the newborn 2 ounces of formula. C. Administer intravenous glucose immediately. D. Initiate blow-by oxygen therapy.

C. Administer intravenous glucose immediately If an LGA newborn's blood glucose level is below 25 mg/dL, the nurse should institute immediate treatment with intravenous glucose regardless of the clinical symptoms. Oral feedings would be used to maintain the newborn's glucose level above 40 mg/dL. Blow-by oxygen would have no effect on glucose levels; it may be helpful in promoting oxygenation. Placing the newborn under a radiant warmer would be a more appropriate measure for cold stress. (ch. 23)

A nurse is assessing a client for possible risk factors for chlamydia and gonorrhea. Which factor would the nurse identify? Answers: A. married B. consistent use of barrier contraception C. age under 25 years D. Asian American ethnicity

C. Age under 25 High-risk groups for chlamydia and gonorrhea include single women, women younger than 25 years, African American women, women with a history of STIs, those with new or multiple sex partners, those with inconsistent use of barrier contraception, and women living in communities with high infection rates. (ch. 5)

After teaching a group of pregnant women about fetal development, the nurse determines that the teaching was successful when the women identify which element as essential for fetal lung development? Answers: A. trophoblasts B. placenta C. amniotic fluid D. umbilical cord

C. Amniotic fluid Amniotic fluid is essential for fetal growth and development, especially fetal lung development. The umbilical cord is the lifeline from the mother to the growing embryo. The placenta serves as the interface between the mother and developing fetus. It secretes hormones and supplies the fetus with nutrients and oxygen needed for growth. The trophoblasts differentiate into all the cells that form that placenta. (ch. 10)

While making rounds in the nursery, the nurse sees a 6-hour-old baby girl gagging and turning bluish. What would the nurse do first? Answers: A. Administer oxygen via facial mask by positive pressure. B. Alert the primary care provider stat, and turn the newborn to her right side. C. Aspirate the oral and nasal pharynx with a bulb syringe. D. Lower the newborn's head to stimulate crying

C. Aspirate the oral and nasal pharynx with a bulb syringe The nurse's first action would be to suction the oral and nasal pharynx with a bulb syringe to maintain airway patency. Turning the newborn to her right side will not alleviate the blockage due to secretions. Administering oxygen via positive pressure is not indicated at this time. Lowering the newborn's head would be inappropriate. (ch. 18)

The nurse is conducting a presentation for a young adult community group about fetal development and pregnancy. The nurse determines that the teaching was successful when the group identifies that the sex of offspring is determined at which time? Answers: A. when the morula forms B. during meiosis cell division C. at fertilization D. during oogenesis

C. At fertilization Sex determination occurs at the time of fertilization. Meiosis refers to cell division resulting in the formation of an ovum or sperm with half the number of chromosomes. The morula develops after a series of four cleavages following the formation of the zygote. Oogenesis refers to the development of a mature ovum, which has half the number of chromosomes. (ch. 10)

When applying the ultrasound transducers for continuous external electronic fetal monitoring, at which location would the nurse place the transducer to record the FHR? Answers: A. over the uterine fundus where contractions are most intense B. above the umbilicus toward the right side of the diaphragm C. between the umbilicus and the symphysis pubis D. between the xiphoid process and umbilicus

C. Between the umbilicus and the symphysis pubis The ultrasound transducer is positioned on the maternal abdomen in the midline between the umbilicus and the symphysis pubis. The tocotransducer is placed over the uterine fundus in the area of greatest contractility. (ch. 14)

When teaching parents about their newborn, the nurse describes the development of a close emotional attraction to a newborn by the parents during the first 30 to 60 minutes after birth. The nurse refers to this process by which term? Answers: A. engrossment B. attachment C. bonding D. reciprocity

C. Bonding The development of a close emotional attraction to the newborn by parents during the first 30 to 60 minutes after birth describes bonding. Reciprocity is the process by which the infant's capabilities and behavioral characteristics elicit a parental response. Engrossment refers to the intense interest during early contact with a newborn. Attachment refers to the process of developing strong ties of affection between an infant and significant other. (ch. 16)

While performing a physical assessment of a newborn boy, the nurse notes diffuse edema of the soft tissues of his scalp that crosses suture lines. The nurse documents this finding as: Answers: A. molding. B. cephalhematoma. C. caput succedaneum. D. microcephaly.

C. Caput succedaneum Caput succedaneum is localized edema on the scalp, a poorly demarcated soft tissue swelling that crosses the suture lines. Molding refers to the elongated shape of the fetal head as it accommodates to the passage through the birth canal. Microcephaly refers to a head circumference that is 2 standard deviations below average or less than 10% of normal parameters for gestational age. Cephalhematoma is a localized effusion of blood beneath the periosteum of the skull. (ch. 18)

The nurse frequently assesses the respiratory status of a preterm newborn based on the understanding that the newborn is at increased risk for respiratory distress syndrome because of which factor? Answers: A. smaller respiratory passages B. immature respiratory control center C. deficiency of surfactant D. inability to clear fluids

C. Deficiency of surfactant A preterm newborn is at increased risk for respiratory distress syndrome (RDS) because of a surfactant deficiency. Surfactant helps to keep the alveoli open and maintain lung expansion. With a deficiency, the alveoli collapse, predisposing the newborn to RDS. An inability to clear fluids can lead to transient tachypnea. Immature respiratory control centers lead to an increased risk for apnea. Smaller respiratory passages led to an increased risk for obstruction. (ch. 24)

The nurse is assessing a 13-year-old girl. Which event would the nurse expect to have occurred first? Answers: A. onset of menses B. evidence of pubic hair C. development of breast buds D. growth spurt

C. Development of breast buds Pubertal events preceding the first menses have an orderly progression beginning with the development of breast buds, followed by the appearance of pubic hair, then axillary hair, then a growth spurt. Menses typically occurs about 2 years after the start of breast development. (ch. 3)

Assessment of a newborn reveals uneven gluteal (buttocks) skin creases and a "clunk" when Ortolani maneuver is performed. What would the nurse suspect? Answers: A. normal newborn variation B. slipping of the periosteal joint C. developmental hip dysplasia D. overriding of the pelvic bone

C. Developmental hip dysplasia A "clunk" indicates the femoral head hitting the acetabulum as the head reenters the area. This, along with uneven gluteal creases, suggests developmental hip dysplasia. These findings are not a normal variation and are not associated with slipping of the periosteal joint or overriding of the pelvic bone. (ch. 18)

Which finding would the nurse expect to find in a client with endometriosis? Answers: A. hot flashes B. fever C. dyspareunia D. fluid retention

C. Dyspareunia The client with endometriosis is often asymptomatic, but clinical manifestations include pain before and during menstrual periods (dyspareunia), pain during or after sexual intercourse, infertility, depression, fatigue, painful bowel movements, chronic pelvic pain, hypermenorrhea, pelvic adhesions, irregular and more frequent menses, and premenstrual spotting. Hot flashes may be associated with premenstrual syndrome or menopause. Fluid retention is associated with premenstrual syndrome. Fever would suggest an infection. (ch. 4)

When developing the plan of care for the parents of a newborn, the nurse identifies interventions to promote bonding and attachment based on the rationale that bonding and attachment are most supported by which measure? Answers: A. expert medical care for the labor and birth B. good nutrition and prenatal care during pregnancy C. early parent-infant contact following birth D. grandparent involvement in infant care after birth

C. Early parent-infant contact following birth Optimal bonding requires a period of close contact between the parents and newborn within the first few minutes to a few hours after birth. Expert medical care, nutrition and prenatal care, and grandparent involvement are not associated with the promotion of bonding. (ch. 16)

A newborn is suspected of developing persistent pulmonary hypertension. The nurse would expect to prepare the newborn for which procedure to confirm the suspicion? Answers: A. chest X-ray B. blood cultures C. echocardiogram D. stool for occult blood

C. Echocardiogram An echocardiogram is used to reveal right-to-left shunting of blood to confirm the diagnosis of persistent pulmonary hypertension. Chest X-ray would be most likely used to aid in the diagnosis of RDS or TTN. Blood cultures would be helpful in evaluating for neonatal sepsis. Stool for occult blood may be done to evaluate for NEC. (ch. 24)

A nurse is developing a plan of care for a woman to ensure continuity of care during pregnancy, labor, and childbirth. Which would be most important for the nurse to incorporate into that plan? Answers: A. Assigning several nurses as a support team B. Adhering to strict, specific routines C. Educating the client about the importance of a support person D. Involving a pediatric physician

C. Educating the client about the importance of a support person Educating the client about the importance of a support person during labor and delivery has been shown to improve and enhance the birthing experience. (ch. 1)

When teaching a group of postmenopausal women about hot flashes and night sweats, the nurse would address which primary cause? Answers: A. changes in vaginal pH B. poor dietary intake C. estrogen deficiency D. active lifestyle

C. Estrogen deficiency Hot flashes and night sweats are classic signs of estrogen deficiency. They are unrelated to dietary intake or active lifestyle. Changes in vaginal pH are associated with genitourinary changes of menopause. (ch. 4)

The nurse is reviewing the physical examination findings for a client who is to undergo labor induction. Which finding would indicate to the nurse that a woman's cervix is ripe in preparation for labor induction? Answers: A. closed B. posterior position C. shortened D. firm

C. Shortened A ripe cervix is shortened, centered (anterior), softened, and partially dilated. An unripe cervix is long, closed, posterior, and firm. (ch. 21)

A newborn with severe meconium aspiration syndrome (MAS) is not responding to conventional treatment. Which measure would the nurse anticipate as possibly necessary for this newborn? Answers: A. insertion of a laryngoscope for deep suctioning B. respiratory support with a ventilator C. extracorporeal membrane oxygenation (ECMO) D. replacement of an endotracheal tube via X-ray

C. Extracorporeal membrane oxygenation (ECMO) If conventional measures are ineffective, then the nurse would need to prepare the newborn for ECMO. Hyperoxygenation, ventilatory support, and suctioning are typically used initially to promote tissue perfusion. However, if these are ineffective, ECMO would be the next step. (ch. 24)

Which finding would the nurse expect to find in a client with bacterial vaginosis? Answers: A. cervical bleeding on contact B. yellowish-green discharge C. fish-like odor of discharge D. vaginal pH of 3

C. Fish-like odor of discharge Manifestations of bacterial vaginosis include a thin, white homogenous vaginal discharge with a characteristic stale fish odor, vaginal pH greater than 4.5, and clue cells on wet-mount examination. A yellowish-green discharge with cervical bleeding on contact would be characteristic of trichomoniasis. (ch. 5)

The nurse is trying to get consent to care for an 11-year-old boy with diabetic ketoacidosis. His parents are out of town on vacation, and the child is staying with a neighbor. Which action would be the priority? Answers: A. contacting the child's aunt or uncle to obtain their consent B. asking the courts to grant permission on the child's behalf C. getting telephone consent with two people listening to the verbal consent D. providing emergency care without parental consent

C. Getting telephone consent with two people listening to the verbal consent The priority action would be to contact the neighbor for an emergency number to reach the parents and get their verbal consent with two witnesses listening simultaneously. If the nurse cannot reach the parents, and there is no relative or other person with written authorization to act on the parent's behalf, then the primary care provider may initiate emergency care without the parents' consent. Involving the court would take too much time. The child needs immediate treatment. (ch. 1)

The nurse is teaching a group of parents who have preterm newborns about the differences between a full-term newborn and a preterm newborn. Which characteristic would the nurse describe as associated with a preterm newborn but not a term newborn? Answers: A. more subcutaneous fat in the neck and abdomen B. fewer visible blood vessels through the skin C. greater surface area in proportion to weight D. well-developed flexor muscles in the extremities

C. Greater surface area in proportion to weight Preterm newborns have large body surface areas compared to weight, which allows an increased transfer of heat from their bodies to the environment. Preterm newborns often have thin transparent skin with numerous visible veins, minimal subcutaneous fat, and poor muscle tone. (ch. 23)

After teaching a group of young adult women on preventing pelvic inflammatory disease, the nurse determines that the teaching was successful when the group identifies which method as effective? Answers: A. using a vaginal douche weekly B. using hormonal contraception C. having infected sexual partners receive treatment D. obtaining routine treatment if at risk and asymptomatic

C. Having infected sexual partners receive treatment Strategies for preventing pelvic inflammatory disease include advising sexually active females to insist their partners use condoms, discouraging routine vaginal douching, encouraging regular sexually transmitted infection screening, and emphasizing the importance of having each sexual partner receive treatment if infected. (ch. 5)

When describing genetic disorders to a group of childbearing couples, the nurse would identify which as an example of an autosomal dominant inheritance disorder? Answers: A. sickle cell disease B. phenylketonuria C. Huntington disease D. cystic fibrosis

C. Huntington Disease Huntington disease is an example of an autosomal dominant inheritance disorder. Sickle cell disease, phenylketonuria, and cystic fibrosis are examples of autosomal recessive inheritance disorders (ch. 10)

A postpartum woman who is breast-feeding tells the nurse that she is experiencing nipple pain. After teaching the woman about possible suggestions, the nurse determines that more teaching is needed when the woman makes which statement? Answers: A. "My baby latches on." B. "I use of a mild analgesic about 1 hour before breast-feeding." C. "I apply glycerin-based gel to my nipples." D. "I apply expressed breast milk to my nipples."

C. I apply a glycerin-based gel to my nipples Nipple pain is difficult to treat, although a wide variety of topical creams, ointments, and gels are available to do so. This group includes beeswax, glycerin-based products, petrolatum, lanolin, and hydrogel products. Many women find these products comforting. Beeswax, glycerin-based products, and petrolatum all need to be removed before breast-feeding. These products should be avoided in order to limit infant exposure because the process of removal may increase nipple irritation. Mild analgesics such as acetaminophen or ibuprofen are considered relatively safe for breast-feeding mothers. Applying expressed breast milk to nipples and allowing it to dry has been suggested to reduce nipple pain. Usually the pain is due to incorrect latch-on and/or removal of the nursing infant from the breast. Early assistance with breast-feeding to ensure correct positioning can help prevent nipple trauma. In addition, applying expressed milk to nipples and allowing it to dry has been suggested to result in less nipple pain for many women. (ch. 16)

A client is diagnosed with fibrocystic breast disease. After teaching the client about this condition, the nurse determines that the teaching was successful based on which client statement? Answers: A. "No more cookies and baked goods for me." B. "It's important that I stop smoking or my condition will get worse." C. "I need to cut out drinking coffee like I'm used to doing." D. "I guess I'll have to find a replacement for milk and cheese."

C. I need to cut out drinking coffee like I'm used to doing Caffeine is a stimulant and eliminating it will help reduce symptoms of fibrocystic breast disease. Thus cutting out coffee from the client's intake indicates understanding of the situation. Although smoking cessation is important for anyone, cigarettes, along with dairy products such as milk and cheese, and sweets, such as cookies and baked goods, are not associated with symptoms of fibrocystic breast disease. (ch. 6)

After assessing a woman who has come to the clinic, the nurse suspects that the woman is experiencing abnormal uterine bleeding. Which statement by the client would support the nurse's suspicions? Answers: A. "I get really irritable and moody about a week before my period." B. "My periods have been unusually long and heavy lately." C. "I've been having bleeding off and on that's irregular and sometimes heavy." D. "I get sharp pain in my lower abdomen usually starting soon after my period comes."

C. I've been having bleeding off and on that's irregular and sometimes heavy Abnormal uterine bleeding is defined as irregular, abnormal bleeding that occurs with no identifiable anatomic pathology. It is frequently associated with anovulatory cycles, which are common for the first year after menarche and later in life as a woman approaches menopause. Pain occurring with menses refers to dysmenorrhea. Although mood swings may be associated with dysfunctional uterine bleeding, irritability and mood swings are more commonly associated with premenstrual syndrome. Unusually long and heavy periods reflect menorrhagia. (ch. 4)

A nurse is teaching a new mother about her newborn's immune status. The nurse determines that the teaching was successful when the mother states which immunoglobulin as having crossed the placenta? Answers: A. IgA B. IgM C. IgG D. IgE

C. IgG IgG is the major immunoglobulin and the most abundant, making up about 80% of all circulating antibodies (Martin, Fanaroff, & Walsh, 2014). It is found in serum and interstitial fluid. It is the only class able to cross the placenta, with active placental transfer beginning at approximately 20 to 22 weeks' gestation. No other immunoglobulin crosses the placenta. (ch. 17)

A nurse is conducting a presentation to a local women's community group about strategies to promote health and wellness and prevent illness. After the presentation, the nurse determines that the teaching was successful based on which statement by the group when describing health? Answers: A. "Health is simply a state of overall wellness." B. "Health is the state of being completely disease free." C. It is complete physical, mental, and social well-being." D. "It is based on analysis of statistics about diseases and deaths."

C. It is complete physical, mental, and social well-being The World Health Organization (WHO, 2015a) defines health as "a state of complete physical, mental, and social well-being, and not merely the absence of disease or infirmity." The definition of health is complex; it is not merely the absence of disease or an analysis of mortality and morbidity statistics. (ch. 1)

A nurse is integrating information about family-centered care when developing a teaching plan for a pregnant woman and her family. Which statement by the woman indicates to the nurse that the teaching was effective? Answers: A. "This is a stressful time for our family, so we won't be making any of the health care decisions." B. "Since I'm the mother, this whole process of childbirth will affect me primarily." C. "It's not just me affected by the baby. My whole family will be affected." D. "Childbirth is a medical problem, so it can affect everyone."

C. It's not just me affected by the baby. My whole family will be affected Childbirth affects the entire family, and relationships will change. Childbirth is viewed as a normal life event, not a medical procedure. Families are very capable of making health care decisions about their own care with proper information and support. (ch. 1)

A nurse is reading a journal article about cesarean births and the indications for them. Which indication for cesarean birth occurs most frequently? Answers: A. fetal malpresentation B. abnormal fetal heart rate tracing C. labor dystocia D. multiple gestation

C. Labor dystocia The most common indications for primary cesarean births include, in order of frequency, labor dystocia, abnormal fetal heart rate tracing, fetal malpresentation, multiple gestation, and super macrosomia. (ch. 21)

The nurse is auscultating a newborn's heart and places the stethoscope at the point of maximal impulse at which location? Answers: A. directly adjacent to the sternum at the second intercostals space B. just superior to the nipple, at the midsternum C. lateral to the midclavicular line at the fourth intercostal space D. at the fifth intercostal space to the left of the sternum

C. Lateral to the midclavicular line at the fourth intercostal space The point of maximal impulse (PMI) in a newborn is a lateral to midclavicular line located at the fourth intercostal space. (ch. 18)

A nurse is assessing a child with Klinefelter syndrome. What would the nurse expect to assess? Select all that apply. Answers: A. enlarged testicles B. gross mental retardation C. long arms D. gynecomastia E. profuse body hair

C. Long arms D. Gynecomastia Manifestations of Klinefelter syndrome include mild mental retardation, small testicles, infertility, long arms and legs, gynecomastia, scant facial and body hair, and decreased libido (ch. 10)

The nurse is developing a plan of care for a woman with breast cancer who is scheduled to undergo breast-conserving surgery. The nurse interprets this as which procedure? Answers: A. entire breast removal without lymph nodes B. axillary lymph node removal C. lump removal followed by radiation D. removal of nipple and areolar area

C. Lump removal followed by radiation Breast-conserving surgery is the wide local excision (or lumpectomy) of the tumor along with a 1-cm margin of normal tissue. A lumpectomy is often used for early-stage localized tumors and is followed by radiation to eradicate residual microscopic cancer cells. A simple mastectomy is the removal of all breast tissue, the nipple, and the areola. The axillary nodes and pectoral muscles are spared. A modified radical mastectomy involves removal of breast tissue, the axillary nodes, and some chest muscles, but not the pectoralis major, thus avoiding a concave anterior chest. (ch. 6)

A client who has come to the clinic is diagnosed with endometriosis. What would the nurse expect the primary care provider to prescribe as a first-line treatment? Answers: A. antiestrogens B. progestins C. NSAIDs D. gonadotropin-releasing hormone analogues

C. NSAIDS Although progestins, antiestrogens, and gonadotrophin-releasing analogues are used as treatment options for endometriosis, NSAIDS are considered the first-line treatment to reduce pain. (ch. 4)

The nurse assesses a 1-day-old newborn. Which finding would the nurse interpret as suggesting an issue with oxygenation? Answers: A. abdominal breathing B. respiratory rate of 54 breaths/minute C. nasal flaring D. acrocyanosis

C. Nasal flaring Nasal flaring is a sign of respiratory difficulty in the newborn. A rate of 54 breaths/minute, diaphragmatic/abdominal breathing, and acrocyanosis are normal findings. (ch. 18)

A nurse is assessing a newborn and observes the newborn moving his head and eyes toward a loud sound. The nurse interprets this as which behavior? Answers: A. social behavior B. motor maturity C. orientation D. habituation

C. Orientation Orientation refers to the response of newborns to stimuli. It reflects newborns' response to auditory and visual stimuli, demonstrated by their movement of head and eyes to focus on that stimulus. Habituation is the newborn's ability to process and respond to visual and auditory stimuli—that is, how well and appropriately he or she responds to the environment. Habituation is the ability to block out external stimuli after the newborn has become accustomed to the activity. Motor maturity depends on gestational age and involves evaluation of posture, tone, coordination, and movements. These activities enable newborns to control and coordinate movement. When stimulated, newborns with good motor organization demonstrate movements that are rhythmic and spontaneous. Social behaviors include cuddling and snuggling into the arms of the parent when the newborn is held. (ch. 17)

After teaching the parents of a newborn with periventricular hemorrhage about the disorder and treatment, which statement by the parents indicates that the teaching was successful? Answers: A. "Once the bleeding ceases, there won't be any more worries." B. "We need to get family members to donate blood for transfusion." C. "Our newborn could develop a learning disability later on." D. "We'll make sure to cover both of his eyes to protect them."

C. Our newborn could develop a learning disability later on Periventricular hemorrhage has long-term sequelae such as seizures, hydrocephalus, periventricular leukomalacia, cerebral palsy, learning disabilities, vision or hearing deficits, and mental retardation. Covering the eyes is more appropriate for the newborn receiving phototherapy. The bleeding in the brain can lead to serious long-term effects. Blood transfusions are not used to treat periventricular hemorrhage. (ch. 24)

A woman with gestational hypertension experiences a seizure. Which intervention would the nurse identify as the priority? Answers: A. control of hypertension B. delivery of the fetus C. oxygenation D. fluid replacement

C. Oxygenation As with any seizure, the priority is to clear the airway and maintain adequate oxygenation both to the mother and the fetus. Fluids and control of hypertension are addressed once the airway and oxygenation are maintained. Delivery of fetus is determined once the seizures are controlled and the woman is stable. (ch. 19)

A nurse is teaching a new mother about breast-feeding. The nurse determines that the teaching was successful when the woman identifies which hormone as responsible for milk let-down? Answers: A. prolactin B. estrogen C. oxytocin D. progesterone

C. Oxytocin Oxytocin is released from the posterior pituitary to promote milk let-down. Prolactin levels increase at term with a decrease in estrogen and progesterone; estrogen and progesterone levels decrease after the placenta is delivered. Prolactin is released from the anterior pituitary gland and initiates milk production. (ch. 15)

Which measure would the nurse include in the teaching plan for a woman to reduce the risk of osteoporosis after menopause? Answers: A. restricting fluid to 1,000 mL daily B. eating high-fiber, high-calorie foods C. participating in regular daily exercise D. taking vitamin supplements

C. Participating in regular daily exercise Measures to reduce osteoporosis after menopause include daily weight-bearing exercise, increasing calcium and vitamin D intake, and avoiding smoking and excessive alcohol intake. General vitamin supplements may be helpful overall, but they are not specific to reducing the risk of osteoporosis. A diet high in calcium and vitamin D, not fiber and calories, would be appropriate. Restricting fluids would have no effect on preventing osteoporosis. (ch. 4)

The nurse is providing care to a newborn with severe meconium aspiration syndrome (MAS). The nurse is reviewing the newborn's diagnostic test results. Which finding would the nurse expect? Answers: A. vocal cords negative for meconium B. elevated blood pH C. patchy fluffy infiltrates on chest X-ray D. increased PaO2

C. Patchy fluffy infiltrates on chest X-ray Chest X-rays show patchy, fluffy infiltrates unevenly distributed throughout the lungs and marked hyperaeration mixed with areas of atelectasis. ABG analysis will indicate metabolic acidosis with a low blood pH, decreased PaO2, and increased PaCO2.Direct visualization of the vocal cords for meconium staining using an appropriate size laryngoscope is needed. (ch. 24)

Which information would the nurse include when teaching a new mother about the difference between pathologic and physiologic jaundice? Answers: A. Both are treated with exchange transfusions of maternal O- blood. B. Physiologic jaundice results in kernicterus. C. Pathologic jaundice appears within 24 hours after birth. D. Physiologic jaundice requires transfer to the NICU.

C. Pathologic jaundice appears within 24 hours after birth Pathologic jaundice appears within 24 hours after birth whereas physiologic jaundice commonly appears around the third to fourth days of life. Kernicterus is more commonly associated with pathologic jaundice. An exchange transfusion is used only if the total serum bilirubin level remains elevated after intensive phototherapy. With this procedure, the newborn's blood is removed and replaced with nonhemolyzed red blood cells from a donor. Physiologic jaundice often is treated at home. (ch. 24)

A nurse is preparing an inservice education program for a group of nurses about dystocia involving problems with the passenger. Which problem would the nurse most likely include as the most common? Answers: A. multifetal pregnancy B. breech presentation C. persistent occiput posterior position D. macrosomia

C. Persistent occiput posterior position Common problems involving the passenger include occiput posterior position, breech presentation, multifetal pregnancy, excessive size (macrosomia) as it relates to cephalopelvic disproportion (CPD), and structural anomalies. Of these, persistent occiput posterior is the most common malposition, occurring in about 15% of laboring women. (ch. 21)

the events of each stage, the nurse determines that the teaching was successful when the women identify which stage? Select all that apply. Answers: A. umbilical B. placental C. preembryonic D. fetal E. embryonic

C. Preembryonic D. Fetal E. Embryonic The three stages of fetal development are the preembryonic, embryonic, and fetal stage. Placental and umbilical are not stages of fetal development. (ch. 10)

A newborn is diagnosed with meconium aspiration syndrome. When assessing this newborn, the nurse would expect which findings? Select all that apply. Answers: A. pigeon chest B. high blood pH level C. prolonged tachypnea D. coarse crackles on auscultation E. intercostal retractions

C. Prolonged tachypnea D. Coarse crackles on auscultation E. Intercostal retractions Assessment findings associated with meconium aspiration syndrome include barrel-shaped chest with an increased anterior-posterior (AP) chest diameter (similar to that found in a client with chronic obstructive pulmonary disease), prolonged tachypnea, progression from mild to severe respiratory distress, intercostal retractions, end-expiratory grunting, and cyanosis. Coarse crackles and rhonchi are noted on lung auscultation. (ch. 24)

The nurse administers vitamin K intramuscularly to the newborn based on which rationale? Answers: A. Stop Rh sensitization. B. Enhance bilirubin breakdown. C. Promote blood clotting. D. Increase erythropoiesis.

C. Promote blood clotting Vitamin K promotes blood clotting by increasing the synthesis of prothrombin by the liver. Rho(D) immune globulin prevents Rh sensitization. Erythropoietin stimulates erythropoiesis. Phototherapy enhances bilirubin breakdown. (ch. 18)

A nurse is developing a plan of care for a preterm newborn to address the nursing diagnosis of risk for delayed development. Which measures would the nurse include? Select all that apply. Answers: A. loosely covering the newborn with blankets B. positioning newborn in extension C. providing nonnutritive sucking D. clustering care to promote rest E. using kangaroo care

C. Providing nonnutritive sucking D Clustering care to promote rest E Using Kangaroo care The nurse would focus the plan of care on developmental care, which includes clustering care to promote rest and conserve energy, using flexed positioning to simulate in utero positioning, using kangaroo care to promote skin to skin sensations, swaddling with a blanket to maintain the flexed position, and providing nonnutritive sucking. (ch. 23)

The partner of a woman who has given birth to a healthy newborn says to the nurse, "I want to be involved, but I'm not sure that I'm able to care for such a little baby." The nurse interprets this as indicating which stage? Answers: A. transition to mastery B. taking-hold C. reality D. expectations

C. Reality The partner's statement reflects stage 2 (reality), which occurs when fathers or partners realize that their expectations in stage 1 are not realistic. Their feelings change from elation to sadness, ambivalence, jealousy, and frustration. Many wish to be more involved in the newborn's care and yet do not feel prepared to do so. New fathers or partners pass through stage 1 (expectations) with preconceptions about what home life will be like with a newborn. Many men may be unaware of the dramatic changes that can occur when this newborn comes home to live with them. In stage 3 (transition to mastery), the father or partner makes a conscious decision to take control and be at the center of his newborn's life regardless of his preparedness. Taking-hold is a stage of maternal adaptation. (ch. 15)

A battered pregnant woman reports to the nurse that her husband has stopped hitting her and promises never to hurt her again. Which response by the nurse would be most appropriate? Answers: A. "That's great. I wish you both the best." B. "You need to consider leaving him." C. "Remember, the cycle of violence often repeats itself." D. "He probably didn't mean to hurt you."

C. Remember, the cycle of violence often repeats itself The cycle of violence typically increases in frequency and severity as it is repeated over and over again. The woman needs to understand this. (ch. 9)

Which instructions would the nurse include when teaching a woman with pediculosis pubis? Answers: A. "Wash your bed linens in bleach and cold water." B. "Take the antibiotic until you feel better." C. "Remove the nits with a fine-toothed comb." D. "Your partner doesn't need treatment at this time."

C. Remove the nits with a fine-toothed comb The nurse should instruct the client to remove the nits from the hair using a fine-toothed comb. Permethrin cream and lindane shampoo are used as treatment, not antibiotics. Bedding and clothing should be washed in hot water to decontaminate it. Sexual partners should be treated also, as well as family members who live in close contact with the infected person. (ch. 5)

A woman in labor received an opioid close to the time of birth. The nurse would assess the newborn for which effect? Answers: A. abdominal distention B. hyperreflexia C. respiratory depression D. urinary retention

C. Respiratory depression Opioids given close to the time of birth can cause central nervous system depression, including respiratory depression, in the newborn, necessitating the administration of naloxone. Urinary retention may occur in the woman who received neuraxial opioids. Abdominal distention is not associated with opioid administration. Hyporeflexia would be more commonly associated with central nervous system depression due to opioids. (ch. 14)

A nurse is conducting a class for a group of nurses new to the labor and birth unit about labor and the passage of the fetus through the birth canal. As part of the class, the nurse explains that specific diameters of the fetal skull can affect the birth process. Which diameter would the nurse identify as being most important? Select all that apply. Answers: A. Occipitomental B. Diagonal conjugate C. Suboccipitobregmatic D. Biparietal E. Occipitofrontal

C. Suboccipitobregmatic D. Biparietal The diameter of the fetal skull is an important consideration during the labor and birth process. Fetal skull diameters are measured between the various landmarks of the skull. Diameters include occipitofrontal, occipitomental, suboccipitobregmatic, and biparietal (Fig. 13.4). The two most important diameters that can affect the birth process are the suboccipitobregmatic (approximately 9.5 cm at term) and the biparietal (approximately 9.25 cm at term) diameters. Diagonal conjugate is a measure of the pelvic inlet of the mother. (ch. 13)

A nurse is teaching postpartum client and her partner about caring for their newborn's umbilical cord site. Which statement by the parents indicates a need for additional teaching? Answers: A. "Exposing the stump to the air helps it to dry." B. "We can put him in the tub to bathe him once the cord falls off and is healed." C. "The cord stump should change from brown to yellow." D. "We need to call the primary care provider if we notice a funny odor."

C. The cord stump should change from brown to yellow The cord stump should change color from yellow to brown or black. Therefore, the parents need additional teaching if they state the color changes from brown to yellow. Tub baths are avoided until the cord has fallen off and the area is healed. Exposing the stump to the air helps it to dry. The parents should notify their primary care provider if there is any bleeding, redness, drainage, or foul odor from the cord stump. (ch. 18)

A nurse is preparing a presentation for a group of neonatal nurses on congenital clubfoot. The nurse determines that the presentation was successful when the group makes which statement? Answers: A. Clubfoot is a common genetic disorder. B. The intrinsic form can be manually reduced. C. The exact cause of clubfoot is not known. D. The condition affects girls more often than boys.

C. The exact cause of clubfoot in not known Clubfoot is a complex, multifactorial deformity with genetic and intrauterine factors. Heredity and race seem to factor into the incidence, but the means of transmission and the etiology are unknown. Most newborns with clubfoot have no identifiable genetic, syndromal, or extrinsic cause. Clubfoot affects boys twice as often as girls. With the intrinsic type, manual reduction is not possible. (ch. 24)

While talking with a pregnant woman who has undergone genetic testing, the woman informs the nurse that her baby will be born with Down syndrome. The nurse understands that Down syndrome is an example of: Answers: A. multifactorial inheritance. B. X-linked recessive inheritance. C. trisomy numeric abnormality. D. chromosomal deletion.

C. Trisomy numeric abnormaility Down syndrome is an example of a chromosomal abnormality involving the number of chromosomes, in particular chromosome 21, in which the individual has three copies of that chromosome. Multifactorial inheritance gives rise to disorders such as cleft lip, congenital heart disease, neural tube defects, and pyloric stenosis. X-linked recessive inheritance is associated with disorders such as hemophilia. Chromosomal deletion is involved with disorders such as cri du chat syndrome. (ch. 10)

A nurse is teaching a pregnant couple about childbirth education. The nurse determines that the teaching was successful when the couple makes which statement? Answers: A. "We'll have the knowledge to ensure a pain-free childbirth." B. "We will be in total control of the birth process." C. "We'll know what to do to actively take part in our child's birth." D. "We won't be anxious, so the birth will be uncomplicated."

C. We'll know what to do to actively take part in our child's birth The primary focus of childbirth education is to provide information and support to clients and their families to foster a more active role in the upcoming birth. Some methods of childbirth education focus on pain-free childbirth. Information provided in childbirth education classes helps to minimize anxiety and provide the couple with control over the situation, but elimination of anxiety or total control is unrealistic. (ch. 12)

The nurse observes the stool of a newborn who is being bottle-fed.The newborn is 2 days old. What would the nurse expect to find? Answers: A. greenish black, tarry stool B. yellow-gold, stringy stool C. yellowish-green, pasty stool D. yellowish-brown, seedy stool

C. Yellowish-green, pasty stool The milk stools of the formula-fed newborn vary depending on the type of formula ingested. They may be yellow, yellow-green, or greenish and loose, pasty, or formed in consistency, and they have an unpleasant odor. After breast-feedings are initiated, a transitional stool develops, which is greenish brown to yellowish brown, thinner in consistency, and seedy in appearance. Meconium stool is greenish black and tarry. The last development in the stool pattern is the milk stool. Milk stools of the breast-fed newborn are yellow-gold, loose, and stringy to pasty in consistency, and typically sour-smelling. (ch. 17)

A biophysical profile has been completed on a pregnant woman. The nurse interprets which score as normal? Answers: A. 7 B. 3 C. 5 D. 9

D. 9 The biophysical profile is a scored test with five components, each worth 2 points if present. A total score of 10 is possible if the NST is used. Overall, a score of 8 to 10 is considered normal if the amniotic fluid volume is adequate. A score of 6 or below is suspicious, possibly indicating a compromised fetus; further investigation of fetal well-being is needed. (ch. 12)

A nurse is conducting a presentation for a group of pregnant women who are considered high-risk. After describing the complications that can occur in newborns, the nurse determines that the teaching was successful when the group identifies which newborn as having the lowest risk for problems? Answers: A. low-birthweight B. large-for-gestational-age C. small-for-gestational-age D. appropriate-for-gestational-age

D. Appropriate-for-gestational age Appropriate-for-gestational-age newborns are at the lowest risk for any problems. The other categories all have an increased risk of complications. (ch. 23)

A client is diagnosed with an enterocele. The nurse interprets this condition as: Answers: A. sagging of the rectum with pressure exerted against the posterior vaginal wall. B. protrusion of the posterior bladder wall downward through the anterior vaginal wall. C. descent of the uterus through the pelvic floor into the vagina. D. bulging of the small intestine through the posterior vaginal wall.

D. Bulging of the small intestine through the posterior vaginal wall An enterocele occurs when the small intestine bulges through the posterior vaginal wall, especially when straining. A cystocele is a protrusion of the posterior bladder wall downward through the anterior vaginal wall. A rectocele occurs when the rectum sags and pushes against or into the posterior vaginal wall. Uterine prolapse occurs when the uterus descends through the pelvic floor and into the vaginal canal. (ch. 7)

A client's membranes spontaneously ruptured, as evidenced by a gush of clear fluid with a contraction. What would the nurse do next? Answers: A. Change the linen saver pad. B. Perform a vaginal exam. C. Notify the primary care provider immediately. D. Check the fetal heart rate.

D. Check the fetal heart rate When membranes rupture, the priority focus is on assessing fetal heart rate first to identify a deceleration, which might indicate cord compression secondary to cord prolapse. A vaginal exam may be done later to evaluate for continued progression of labor. The primary care provider should be notified, but this is not a priority at this time. Changing the linen saver pad would be appropriate once the fetal status is determined and the primary care provider has been notified. (ch. 14)

A group of nurses are attending an in-service program on STIs. The group demonstrates understanding of the information when they identify which STI as the most common bacterial STI in the United States? Answers: A. candidiasis B. syphilis C. gonorrhea D. chlamydia

D. Chlamydia According to the CDC, chlamydia is the most common bacterial STI in the United States. Gonorrhea and syphilis are bacterial infections but not the most common ones. Candidiasis is a fungal infection. (ch. 5)

A postpartum client has a fourth-degree perineal laceration. The nurse would expect which medication to be ordered? Answers: A. bromocriptine B. ferrous sulfate C. methylergonovine D. docusate

D. Docusate A stool softener such as docusate may promote bowel elimination in a woman with a fourth-degree laceration, who may fear that bowel movements will be painful. Ferrous sulfate would be used to treat anemia. However, it is associated with constipation and would increase the discomfort when the woman has a bowel movement. Methylergonovine would be used to prevent or treat postpartum hemorrhage. Bromocriptine is used to treat hyperprolactinemia (ch. 16)

The plan of care for a woman diagnosed with a suspected reproductive cancer includes a nursing diagnosis of disturbed body image related to suspected reproductive tract cancer and impact on sexuality as evidenced by the client's statement that she is worried that she will not be the same. Which outcome would be appropriate for this client? Answers: A. Client will identify misconceptions related to her diagnosis. B. Client will demonstrate understanding of the condition and associated treatment. C. Client will exhibit positive coping strategies related to diagnosis. D. Client will verbalize positive statements about self and sexuality.

D. Client will verbalize positive statements about self and sexuality An appropriate outcome for disturbed body image would be that the client verbalizes positive statements about herself and her sexuality. Demonstrating understanding of the condition and treatment and identifying misconceptions would be appropriate for a nursing diagnosis of deficient knowledge. Exhibiting positive coping strategies would be appropriate for a nursing diagnosis of anxiety. (ch. 8)

A group of students are reviewing an article describing information related to indicators for women's health and the results of a national study. Which would the students identify as being satisfactory for women? Select all that apply. Answers: A. Smoking cessation B. Health insurance coverage C. Violence against women D. Colorectal cancer screening E. Mammograms

D. Colorectal cancer screening E. Mammograms In the 2010 National Report Card, the nation met three benchmarks since 2007: colorectal cancer screening, annual dental visit, and mammograms. All other areas were considered unsatisfactory. (ch. 1)

After teaching parents about their newborn, the nurse determines that the teaching was successful when they identify which concept as reflecting the enduring nature of their relationship involving placing the infant at the center of their lives and finding their own way to assume the parental identity? Answers: A. attachment B. bonding C. reciprocity D. commitment

D. Commitment Commitment refers to the enduring nature of the relationship. The components of this are twofold: centrality and parent role exploration. In centrality, parents place the infant at the center of their lives. They acknowledge and accept their responsibility to promote the infant's safety, growth, and development. Parent role exploration is the parents' ability to find their own way and integrate the parental identity into themselves. The development of a close emotional attraction to the newborn by parents during the first 30 to 60 minutes after birth describes bonding. Reciprocity is the process by which the infant's capabilities and behavioral characteristics elicit a parental response. Engrossment refers to the intense interest during early contact with a newborn. Attachment refers to the process of developing strong ties of affection between an infant and significant other. (ch. 16)

The nurse places a warmed blanket on the scale when weighing a newborn to minimize heat loss via which mechanism? Answers: A. radiation B. evaporation C. convection D. conduction

D. Conduction Using a warmed cloth diaper or blanket to cover any cold surface, such as a scale, that touches a newborn directly helps to prevent heat loss through conduction. Drying a newborn and promptly changing wet linens, clothes, or diapers help reduce heat loss via evaporation. Keeping the newborn out of a direct cool draft, working inside an isolette as much as possible, and minimizing the opening of portholes help prevent heat loss via convection. Keeping cribs and isolettes away from outside walls, cold windows, and air conditioners and using radiant warmers while transporting newborns and performing procedures will help reduce heat loss via radiation. (ch. 17)

A 10-week pregnant woman with diabetes has a glycosylated hemoglobin (HbA1c) level of 13%. At this time the nurse should be most concerned about which possible fetal outcome? Answers: A. incompetent cervix B. placenta previa C. abruptio placentae D. congenital anomalies

D. Congenital anomalies A HbA1c level of 13% indicates poor glucose control. This, in conjunction with the woman being in the first trimester, increases the risk for congenital anomalies in the fetus. Elevated glucose levels are not associated with incompetent cervix, placenta previa, or abruptio placentae. (ch. 20)

The daughter of a woman who has been diagnosed with ovarian cancer asks the nurse about screening for this cancer. Which response by the nurse would be most appropriate? Answers: A. "A genetic test for two genes, if positive, will identify the ovarian cancer." B. "There's a blood test for a marker, CA-125, that if elevated indicates cancer." C. "A Pap smear is almost always helpful in identifying this type of cancer." D. "Currently there is no reliable screening test for ovarian cancer."

D. Currently there is no reliable screening test for ovarian cancer Currently there are no adequate screening tests for ovarian cancer. A Pap smear is used to screen for cervical cancer. The CA-125 marker may be elevated in women with ovarian cancer, but it is not specific for this cancer and may be elevated in other malignancies. Genetic testing via BRCA-1 and BRCA-2 provides information about a woman's risk but does not predict if the woman will develop cancer. (ch. 8)

A nurse is preparing a couple and their newborn for discharge. Which instructions would be most appropriate for the nurse include in discharge teaching? Answers: A. introducing solid foods immediately to increase sleep cycle B. encouraging daily outings to the shopping mall with the newborn C. allowing the infant to cry for at least an hour before picking him or her up D. demonstrating comfort measures to quiet a crying infant

D. Demonstrating comfort measures to quiet a crying infant Discharge teaching typically would focus on several techniques to comfort a crying newborn. The nurse needs to emphasize the importance of responding to the newborn's cues, not allowing the infant to cry for an hour before being comforted. Information about solid foods is inappropriate for a newborn because solid foods are not introduced at this time. The mother and newborn need rest periods. Therefore, daily outings to a shopping mall would be inappropriate. Information about newborn sleep-wake cycles and measures for sensory enrichment and stimulation would be more appropriate. (ch. 16)

A woman who is 42 weeks pregnant comes to the clinic. During the visit, which assessment would be most important for the nurse to perform? Answers: A. measuring the height of the fundus B. checking for spontaneous rupture of membranes C. asking her about the occurrence of contractions D. determining an accurate gestational age

D. Determining an accurate gestational age Incorrect dates account for the majority of postterm pregnancies; many women have irregular menses and thus cannot identify the date of their last menstrual period accurately. Therefore, accurate gestational dating via ultrasound is essential. Asking about contractions and checking for ruptured membranes, although important assessments, would be done once the gestational age is confirmed. Measuring the height of the fundus would be unreliable because after 36 weeks, the fundal height drops due to lightening and may no longer correlate with gestational weeks. (ch. 21)

A 42-year-old woman is scheduled for a mammogram. Which statement would the nurse include when teaching the woman about the procedure? Answers: A. "The room will be darkened throughout the procedure." B. "Make sure to refrain from eating or drinking after midnight." C. "A dye will be injected to highlight the breast tissue and its ducts." D. "Each breast will be firmly compressed between two plates."

D. Each breast will be firmly compressed between two plates A mammogram involves taking x-ray pictures of the breasts while they are compressed between two plastic plates. There is no need to darken the room or to refrain from eating or drinking after midnight. A ductography involves the injection of dye to highlight the breast ducts. (ch. 6)

The nurse is reviewing the laboratory test results of a pregnant client. Which finding would alert the nurse to the development of HELLP syndrome? Answers: A. leukocytosis B. elevated platelet count C. hyperglycemia D. elevated liver enzymes

D. Elevated liver enzymes HELLP is an acronym for hemolysis, elevated liver enzymes, and low platelets. Hyperglycemia or leukocytosis is not a part of this syndrome. (ch. 19)

A nurse is making a home visit to a postpartum woman who delivered a healthy newborn 4 days ago. The woman's breasts are swollen, hard, and tender to the touch. The nurse documents this finding as: Answers: A. involution. B. mastitis. C. engrossment. D. engorgement.

D. Engorgement Engorgement is the process of swelling of the breast tissue as a result of an increase in blood and lymph supply as a precursor to lactation (Figure 15.4). Breast engorgement usually peaks in 3 to 5 days postpartum and usually subsides within the next 24 to 36 hours (Chapman, 2011). Engorgement can occur from infrequent feeding or ineffective emptying of the breasts and typically lasts about 24 hours. Breasts increase in vascularity and swell in response to prolactin 2 to 4 days after birth. If engorged, the breasts will be hard and tender to touch. Involution refers to the process of the uterus returning to its prepregnant state. Mastitis refers to an infection of the breasts. Engrossment refers to the bond that develops between the father and the newborn. (ch. 15)

Assessment of a woman in labor who is experiencing hypertonic uterine dysfunction would reveal contractions that are: Answers: A. brief. B. poor in quality. C. well coordinated. D. erratic.

D. Erratic Hypertonic contractions occur when the uterus never fully relaxes between contractions, making the contractions erratic and poorly coordinated because more than one uterine pacemaker is sending signals for contraction. Hypotonic uterine contractions are poor in quality, brief, and lack sufficient intensity to dilate and efface the cervix. (ch. 21)

A nurse is conducting an in-service program for a group of staff nurses working at the women's health center. After teaching the group about genital fistulas, the nurse determines that the teaching was successful when the group identifies which as a major cause of genital fistulas? Answers: A. congenital anomaly B. Bartholin's gland abscess C. radiation therapy D. female genital cutting

D. Female genital cutting Although genital fistulas may be due to radiation therapy, congenital anomaly, or Bartholin's gland abscess, the majority of fistulas are caused by obstetric trauma and female genital cutting. (ch. 7)

The nurse is explaining fetal circulation to a pregnant woman during an early prenatal visit. The nurse emphasizes the difference in her baby's circulation from the woman's circulation. The nurse determines that the teaching was successful when the woman describes which reason for the difference? Answers: A. Fetal blood is thicker than that of adults and needs different pathways. B. Fetal blood has a higher concentratioin of oxygen and circulates more slowly. C. Fetal heart rates are rapid and circulation time is double that of adults. D. Fetal circulation carries rich, oxygenated blood to vital areas first.

D. Fetal circulation carries rich, oxygenated blood to vital areas first Fetal circulation functions to carry highly oxygenated blood to vital areas first while shunting it away from less vital ones. Fetal blood is not thicker than that of adults. Large volumes of oxygenated blood are not needed because the placenta essentially takes over the functions of the lung and liver during fetal life. Although fetal heart rates normally range from 120 to 160 beats per minute, circulation time is not doubled. (ch. 10)

A client is scheduled for cryosurgery to remove some abnormal tissue on the cervix. The nurse teaches the client about this treatment, explaining that the tissue will be removed by which method? Answers: A. burning B. cutting C. irradiating D. freezing

D. Freezing Feedback: Cryosurgery destroys abnormal cervical tissue by freezing. Conization involves cutting out a cone-shaped section of tissue. Laser therapy destroys cervical tissue by using high-energy light to burn it off. Radiation therapy involves irradiating the tissue for destruction. (ch. 8)

A client is admitted in the healthcare facility with pelvic inflammatory disease (PID). When reviewing the client's history, which of the following would the nurse identify as a risk factor? Answers: A. Environmental exposure B. Genetic predisposition C. Gestational diabetes D. Frequent douching

D. Frequent douching One of the risk factors associated with pelvic inflammatory disease is frequent douching. Women with gestational diabetes are at an increased risk for developing type 2 diabetes later in life. Genetic predisposition and environmental exposure are risk factors associated with breast cancer. (ch. 5)

A woman at 10 weeks gestation comes to the clinic for an evaluation. Which finding might lead the nurse to suspect gestational trophoblastic disease? Answers: A. blood pressure of 120/84 mm Hg B. report of frequent mild nausea C. history of bright red spotting 6 weeks ago D. fundal height measurement of 18 cm

D. Fundal height measurement of 18 cm Findings with gestational trophoblastic disease (hydatidiform mole) may include uterine size larger than expected. Mild nausea would be a normal finding at 10 weeks' gestation. Blood pressure of 120/84 would not be associated with hydatidiform mole and depending on the woman's baseline blood pressure may be within acceptable parameters for her. Bright red spotting might suggest a spontaneous abortion. (ch. 19)

A nurse is conducting a class for pregnant women who are in their third trimester. The nurse is reviewing information about the emotional changes that occur in the postpartum period, including postpartum blues and postpartum depression. After reviewing information about postpartum blues, the group demonstrates understanding when they make which statement about this condition? Answers: A. "Postpartum blues is a long-term emotional disturbance." B. "Extended psychotherapy is needed for treatment." C. "The mother loses contact with reality." D. "Getting some outside help for housework can lessen feelings of being overwhelmed."

D. Getting some outside help for housework can lessen feelings of being overwhelmed Postpartum blues require no formal treatment other than support and reassurance because they do not usually interfere with the woman's ability to function and care for her infant. Nurses can ease a mother's distress by encouraging her to vent her feelings and by demonstrating patience and understanding with her and her family. Suggest that getting outside help with housework and infant care might help her to feel less overwhelmed until the blues ease. Provide telephone numbers she can call when she feels down during the day. Making women aware of this disorder while they are pregnant will increase their knowledge about this mood disturbance, which may lessen their embarrassment and increase their willingness to ask for and accept help if it does occur. (ch. 16)

The nurse is working with a group of community health members to develop a plan to address the special health needs of women. The group would design educational programs to address which condition as the priority? Answers: A. cancer B. smoking C. diabetes D. heart disease

D. Heart disease The group needs to address cardiovascular disease, the number one cause of death in women regardless of racial or ethnic group. Smoking is related to heart disease and the development of cancer. However, heart disease and cancer can occur in any woman regardless of her smoking history. Cancer is the second leading cause of death, with women having a one in three lifetime risk of developing cancer. Diabetes is another important health condition that can affect women. However, it is not the major health problem that heart disease is. (ch. 1)

After teaching a couple about what to expect with their planned cesarean birth, which statement indicates the need for additional teaching? Answers: A. "I guess the nurses will be getting me up and out of bed rather quickly." B. "I'll probably have a tube in my bladder for about 24 hours or so." C. "Holding a pillow against my incision will help me when I cough." D. "I'm going to have to wait a few days before I can start breast-feeding."

D. I'm going to have to wait a few days before I can start breast-feeding Typically, breast-feeding is initiated early as soon as possible after birth to promote bonding. The woman may need to use alternate positioning techniques to reduce incisional discomfort. Splinting with pillows helps to reduce the discomfort associated with coughing. Early ambulation is encouraged to prevent respiratory and cardiovascular problems and promote peristalsis. An indwelling urinary catheter is typically inserted to drain the bladder. It usually remains in place for approximately 24 hours. (ch. 21)

A pregnant woman has a rubella titer drawn on her first prenatal visit. The nurse explains that this test measures: Answers: A. Rh status. B. red blood cell count. C. platelet level. D. immunity to German measles.

D. Immunity to German measles A rubella titer detects antibodies for the virus that causes German measles. If the titer is 1:8 or less, the woman is not immune and requires immunization after birth. Platelet level and red blood cell count would be determined by a complete blood count. Rh status would be determined by blood typing. (ch. 12)

A client asks the nurse, "I read an article on the internet that was talking about integrative medicine. What is that?" Which response by the nurse would be most appropriate? Answers: A. "It means that complementary and alternative medicine is used together with conventional therapies to reduce pain or discomfort." B. "It refers to situations when a client and his or her family prefer to use an unproven method of treatment over a proven one." C. "It refers to the use of complementary and alternative medicine in place of traditional therapies for a condition." D. "It means that mainstream medical therapies and complementary and alternative therapies are combined based on scientific evidence for being effective."

D. It means that mainstream medical therapies and complementary and alternative therapies are combined based on scientific evidence for being effective Integrative medicine combines mainstream medical therapies and CAM therapies for which there is some scientific evidence of safety and effectiveness (NCCAM, 2011). These include acupuncture, reflexology, therapeutic touch, meditation, yoga, herbal therapies, nutritional supplements, homeopathy, naturopathic medicine, and many more used for the promotion of health and well-being. Complementary medicine is used together with conventional medicine, such as using aromatherapy to reduce discomfort after surgery or to reduce pain during a procedure or during early labor. Alternative medicine is used in place of conventional medicine, such as eating a special natural diet to control nausea and vomiting or to treat cancer instead of undergoing surgery, chemotherapy, or radiation that has been recommended by a conventional doctor. (ch. 2)

A nurse is teaching a pregnant woman with preterm premature rupture of membranes who is about to be discharged home about caring for herself. Which statement by the woman indicates a need for additional teaching? Answers: A. "I need to call my doctor if my temperature increases." B. "I need to keep a close eye on how active my baby is each day." C. "I can shower, but I shouldn't take a tub bath." D. "It's okay for my husband and me to have sexual intercourse."

D. It's okay for my husband and me to have sexual intercourse The woman with preterm premature rupture of membranes should monitor her baby's activity by performing fetal kick counts daily, check her temperature and report any increases to the health care provider, not insert anything into her vagina or vaginal area, such as tampons or vaginal intercourse, and avoid sitting in a tub bath. (ch. 19)

A mother brings her 12-year old daughter to the clinic for a visit. The daughter has just started menstruating. As part of the visit, the nurse explains the menstrual cycle and ovulation, including the events that led up to ovulation. Which hormone would the nurse identify as being primarily responsible for ovulation? Answers: A. estrogen B. follicle-stimulating hormone C. progesterone D. luteinizing hormone

D. Luteinizing hormone At ovulation, a mature follicle ruptures in response to a surge of luteinizing hormone. Estrogen is predominant at the end of the follicular phase, directly preceding ovulation. Progesterone peaks 5 to 7 days after ovulation. Follicle-stimulating hormone is highest during the first week of the follicular phase of the cycle. (ch. 3)

The health care provider orders PGE2 for a woman to help evacuate the uterus following a spontaneous abortion. Which action would be most important for the nurse to do? Answers: A. Administer intramuscularly into the deltoid area. B. Use clean technique to administer the drug. C. Keep the gel cool until ready to use. D. Maintain the client supine for 1/2 hour after administration.

D. Maintain the client supine for 1/2 hour after administration When PGE2 is ordered, the gel should come to room temperature before administering it. Sterile technique should be used, and the client should remain supine for 30 minutes after administration. RhoGAM is administered intramuscularly into the deltoid area. (ch. 19)

A nurse is describing X-linked recessive disorders to a group of women undergoing genetic testing. The nurse determines that the teaching was successful when the group states which information? Answers: A. Daughters are more commonly affected with the disorder. B. Males are typically carriers of the disorders. C. Both sons and daughters have a 50% risk of the disorder. D. No male-to-male transmission occurs.

D. No male-to-male transmission occurs Most X-linked disorders demonstrate a recessive pattern of inheritance. Males are more affected than females. A male has only one X chromosome, and all the genes on his X chromosome will be expressed whereas a female will usually need both X chromosomes to carry the disease. There is no male-to-male transmission (since no X chromosome from the male is transmitted to male offspring), but any man who is affected will have carrier daughters. If a woman is a carrier, there is a 50% chance that her sons will be affected and a 50% chance that her daughters will be carriers. (ch. 10)

The nurse is assessing a 12-year-old girl who has had her first menses. When reviewing the client's history, which event would the nurse expect to have most likely occurred last? Answers: A. growth spurt B. breast bud development C. evidence of pubic hair D. onset of menses

D. Onset of menses Pubertal events preceding the first menses have an orderly progression beginning with the development of breast buds, followed by the appearance of pubic hair, then axillary hair, then a growth spurt. Menses typically occurs about 2 years after the start of breast development. (ch. 3)

A 58-year-old client comes to the clinic for evaluation. After obtaining the client's history, the nurse suspects endometrial cancer. Which information would lead the nurse to suspicion? Answers: A. menopause occurring at age 46 B. use of intrauterine device for 3 years C. use of oral contraceptives between ages 18 and 25 D. onset of painless, red postmenopausal bleeding

D. Onset of painless, red postmenopausal bleeding Any episode of bright red painless bleeding occurring after menopause needs to be investigated. Abnormal uterine bleeding in postmenopausal women should be regarded with suspicion. Oral contraceptive use is associated with cervical cancer. Late menopause (after age 52) is associated with endometrial cancer. Use of an intrauterine device is not associated with endometrial cancer. (ch. 8)

A woman comes to the clinic for an evaluation. During the visit, the woman tells the nurse that her menstrual cycles have become irregular. "I've also been waking up at night feeling really hot and sweating. The nurse interprets these findings as: Answers: A. menarche. B. climacteric. C. menopause. D. perimenopause

D. Perimenopause Perimenopause is the time period occurring 2 to 8 years prior to menopause during which women may experience physical changes associated with decreasing estrogen levels, which may include vasomotor symptoms of hot flashes, irregular menstrual cycles, sleep disruptions, forgetfulness, irritability, mood disturbances, decreased vaginal lubrication, night sweats, fatigue, vaginal atrophy, and depression (Burbos & Morris, 2011). Vasomotor symptoms (hot flushes and night sweats) are the most common complaints for which women seek treatment. Menopause or climacteric is defined as 1 year without a menstrual period. Menarche refers to the onset of the first menses. (ch. 3)

A postpartum woman is having difficulty voiding for the first time after giving birth. Which action would be least effective in helping to stimulate voiding? Answers: A. standing her in the shower with the warm water on B. pouring warm water over her perineal area C. having her hear the sound of water running nearby D. placing her hand in a basin of cool water

D. Placing her hand in a basin of cool water Helpful measures to stimulate voiding include placing her hand in a basin of warm water, pouring warm water over her perineal area, hearing the sound of running water nearby, blowing bubbles through a straw, standing in the shower with the warm water turned on, and drinking fluids. (ch. 16)

In the menstrual cycle, every month the female reproductive system generates an ovum. When the ovum is not fertilized, production of which of the following leads to menstruation? Answers: A. Another ovum which begins immediately. B. Follicle stimulating hormone by the anterior pituitary. C. Estrogen by the corpus luteum begins to decrease. D. Progesterone by the corpus luteum beginning to decrease.

D. Progesterone by the corpus luteum beginning to decrease If the ovum is not fertilized, the production of progesterone by the corpus luteum begins to decrease and it changes from a yellow to a white spot on the ovary (corpus albicans). Without the high level of progesterone, the endometrium degenerates and sheds, a process referred to as menstruation. The corpus luteum does not produce estrogen. The production of another ovum is not the factor that causes endometrium degeneration. Production of follicle stimulating hormone by the anterior pituitary initiates ovulation.

While talking with a woman in her third trimester, which behavior indicates to the nurse that the woman is learning to give of oneself? Answers: A. longing to hold infant B. unconditionally accepting the pregnancy without rejection C. showing concern for self and fetus as a unit D. questioning ability to become a good mother

D. Questioning ability to become a good mother Learning to give of oneself would be demonstrated when the woman questions her ability to become a good mother to the infant. Showing concern for herself and fetus as a unit reflects the task of ensuring safe passage throughout pregnancy and birth. Unconditionally accepting the pregnancy reflects the task of seeking acceptance of the infant by others. Longing to hold the infant reflects the task of seeking acceptance of self in the maternal role to the infant (ch. 11)

Just after delivery, a newborn's axillary temperature is 94° F (34.4° C). What action would be most appropriate? Answers: A. Assess the newborn's gestational age. B. Notify the primary care provider if the temperature goes lower. C. Observe the newborn every hour. D. Rewarm the newborn gradually.

D. Rewarm the newborn gradually A newborn's temperature is typically maintained at 97.7° F to 99.7° F (36.5° C to 37.5° C). Since this newborn's temperature is significantly lower, the nurse should institute measures to rewarm the newborn gradually. Assessment of gestational age is completed regardless of the newborn's temperature. Observation would be inappropriate because lack of action may lead to a further lowering of the temperature. The nurse should notify the primary care provider of the newborn's current temperature since it is outside normal parameters. (ch. 18)

When assessing a newborn's reflexes, the nurse strokes the newborn's cheek, and the newborn turns toward the side that was stroked and begins sucking. The nurse documents which reflex as being positive? Answers: A. tonic neck reflex B. palmar grasp reflex C. Moro reflex D. rooting reflex

D. Rooting reflex The rooting reflex is elicited by stroking the newborn's cheek. The newborn should turn toward the side that was stroked and should begin to make sucking movements. The palmar grasp reflex is elicited by placing a finger on the newborn's open palm. The baby's hand will close around the finger. Attempting to remove the finger causes the grip to tighten. The tonic neck reflex is elicited by having the newborn lie on the back and turning the head to one side. The arm toward which the baby is facing should extend straight away from the body with the hand partially open, whereas the arm on the side away from the face is flexed and the fist is clenched tightly. Reversing the direction to which the face is turned reverses the position. The Moro reflex is elicited by placing the newborn on his or her back, supporting the upper body weight of the supine newborn by the arms using a lifting motion without lifting the newborn off the surface. The arms are released suddenly, the newborn will throw the arms outward and flex the knees, and then the arms return to the chest. The fingers also spread to form a C. (ch. 18)

A nurse manager is interviewing a potential candidate for employment in the perinatal clinic. During the interview, the nurse manager asks the nurse about her understanding of family-centered care. Which statement by the interviewee demonstrates understanding of this concept? Answers: A. "Mothers are the family members affected by childbirth." B. "Families lack the ability to make health care decisions for themselves." C. "Childbirth rarely affects the relationships within a family." D. "Sensitivity to and respect for the family's culture is important."

D. Sensitivity to and respect for the family's culture is important Family-centered care requires the nurse to apply sensitivity to the client's and family's beliefs and those supporting their culture. This involves listening to the family's needs and shifting the nurse's authoritarian role to the family to empower them to make their own decisions within the context of a supportive environment. A true family perspective should be applied in maternity care and the new parents viewed as a family unit, not as a medical case. Childbirth affects the entire family, not just the mother, and relationships will change. Childbirth is viewed as a normal life event, not a medical procedure. Families are capable of making health care decisions about their own care with information and support. (ch. 1)

After teaching a group of adolescents about HIV, the nurse asks them to identify the major means by which adolescents are exposed to the virus. The nurse determines that the teaching was successful when the group identifies which means of exposure? Answers: A. sharing needles for IV drug use B. perinatal transmission C. blood transfusion D. sexual intercourse

D. Sexual intercourse Most HIV-infected adolescents are exposed to the virus through sexual intercourse, with recent data suggesting that the majority of HIV-infected adolescent males are infected through sex with men. Only a few adolescent males appear to be exposed through injection of drugs or heterosexual contact. Adolescent females are mostly exposed through heterosexual contact, with a small percentage exposed through injected drug use. Although perinatal transmission can occur, it is not the major means of transmission for adolescents. Exposure to the virus via blood transfusions had dropped significantly with the testing of blood and blood products. (ch. 5)

A woman with polycystic ovary syndrome tells the nurse, "I hate this disease. Just look at me! I have no hair on the front of my head but I've got hair on my chin and upper lip. I don't feel like a woman anymore." Further assessment reveals breast atrophy and increased muscle mass. Which nursing diagnosis would the nurse identify as the priority? Answers: A. ineffective peripheral tissue perfusion related to effects of disease on vasculature B. social isolation related to feelings about appearance C. risk for suicide related to effects of condition and fluctuating hormone levels D. situational low self-esteem related to masculinization effects of the disease

D. Situational low self-esteem related to masculinization effects of the disease The woman is verbalizing how she sees herself in light of the manifestations of PCOS. She is exhibiting a negative self-image. Therefore, the nursing diagnosis of situational low self-esteem would be a priority. There is no information about the woman's participation in social activities. Her statements do not reflect that she might hurt herself. PCOS is associated with long-term health problems, but this is not evidenced by the scenario. (ch. 7)

A client with advanced breast cancer, who has had both chemotherapy and radiation therapy, is to start endocrine therapy. Which agent would the nurse expect the client to receive? Answers: A. trastuzumab B. cortisone C. estrogen D. tamoxifen

D. Tamoxifen The objective of endocrine therapy is to block or counter the effect of estrogen in the pathogenesis of cancer. The best-known agent is tamoxifen. Use of estrogens in postmenopausal women increases a woman's risk for breast cancer. In addition, estrogen is a considered to play a major role in the development of breast cancer and as such would not be used. Cortisone is a steroid and would not be used. Trastuzumab is an immunotherapeutic agent. (ch. 6)

A nurse is conducting a refresher program for a group of nurses about chemotherapy used for breast cancer. After teaching the group about the different types of chemotherapeutic agents used to treat breast cancer, the nurse determines that the teaching was successful when the group identifies which agent as an example of a selective estrogen receptor modulator (SERM)? Select all that apply. Answers: A. exemestane B. anastrozole C. letozole D. tamoxifen E. raloxifene

D. Tamoxifen E. Raloxifene Examples of SERMs include tamoxifen and raloxifene. Letozole, exemestane, and anastrozole are examples of aromatase inhibitors. (ch. 6)

When reviewing the medical record of a postpartum client, the nurse notes that the client has experienced a third-degree laceration. The nurse understands that the laceration extends to which area? Answers: A. through the perineal muscles B. superficial structures above the muscle C. through the anterior rectal wall D. through the anal sphincter muscle

D. Through the anal sphincter muscle A third-degree laceration extends through the anal sphincter muscle. A first-degree laceration involves only the skin and superficial structures above the muscle. A second-degree laceration extends through the perineal muscles. A fourth-degree laceration continues through the anterior rectal wall. (ch. 16)

A primipara client gave birth vaginally to a healthy newborn girl 48 hours ago. The nurse palpates the client's fundus and documents which finding as normal? Answers: A. four fingerbreadths below the umbilicus B. at the level of the umbilicus C. two fingerbreadths above the umbilicus D. two fingerbreadths below the umbilicus

D. Two fingerbreadths below the umbilicus During the first few days after birth, the uterus typically descends downward from the level of the umbilicus at a rate of 1 cm (1 fingerbreadth) per day so that by day 2, it is about 2 fingerbreadths below the umbilicus. (ch. 15)

The nurse is reviewing the process of oocyte maturation and ovulation with a client. What occurs during the follicular phase of the ovarian cycle that the nurse should include in the teaching session on ovulation with the client? Answers: A. Preparation of the uterus for implantation of an ovum B. About day 14, a surge of hormones cause the ovum to burst through the ovary C. The empty ruptured graafian follicle becomes the corpus luteum and it begins to secrete progesterone and estrogen D. Under the influence of follicle-stimulating hormone, several follicles begin to ripen, and the ovum with each begins to mature

D. Under the influence of follicle-stimulating hormone, several follicles begin to ripen, and the ovum with each begins to mature The follicular phase lasts from about day 4 to about day 14. During this time, under the influence of FSH, several follicles begin to ripen and the ovum within each begins to mature. About day 14, a surge of hormones causes the ovum to burst through the ovary; this act is called ovulation. During the luteal phase, the empty, ruptured graafian follicle becomes the corpus luteum and it begins to secrete progesterone and estrogen. The endometrium of the uterus has a similar cycle. It is called the uterine cycle or endometrial cycle. This process prepares the uterus for implantation of an ovum (egg). (ch. 3)

Which female reproductive tract structure would the nurse describe to a group of young women as containing rugae that enable it to dilate during labor and birth? Answers: A. fallopian tube B. vulva C. cervix D. vagina

D. Vagina The vagina is a tubular, fibromuscular organ lined with mucous membrane that lies in a series of transverse folds called rugae. These rugae allow for extreme dilation of the canal during labor and birth. The cervix, the lower portion of the uterus, is composed of fibrous connective tissue that dilates during labor. The fallopian tube transports the ovum from the ovary to the uterus. The vulva is a collective term used to refer to the external female reproductive organs (mons pubis, labia majora and minora, clitoris, vestibular structures, and perineum). (ch. 3)

A nurse is describing the different types of regional analgesia and anesthesia for labor to a group of pregnant women. Which statement by the group indicates that the teaching was successful? Answers: A. "Higher anesthetic doses are needed for patient-controlled epidural analgesia." B. "A pudendal nerve block is highly effective for pain relief in the first stage of labor." C. "Local infiltration using lidocaine is an appropriate method for controlling contraction pain." D. "We can get up and walk around after receiving combined spinal-epidural analgesia."

D. We can get up and walk around after receiving combined spinal-epidural analgesia When compared with traditional epidural or spinal analgesia, which often keeps the woman lying in bed, combined spinal-epidural analgesia allows the woman to ambulate ("walking epidural"). Patient-controlled epidural analgesia provides equivalent analgesia with lower anesthetic use, lower rates of supplementation, and higher client satisfaction. Pudendal nerve blocks are used for the second stage of labor, an episiotomy, or an operative vaginal birth with outlet forceps or vacuum extractor. Local infiltration using lidocaine does not alter the pain of uterine contractions, but it does numb the immediate area of the episiotomy or laceration. (ch. 14)

A woman gives birth to a newborn at 36 weeks' gestation. She tells the nurse, "I'm so glad that my baby isn't premature." Which response by the nurse would be most appropriate? Answers: A. "You are lucky to have given birth to a term newborn." B. "How do you feel about delivering your baby at 36 weeks?" C. "Your baby is premature and needs monitoring in the NICU." D. "We still need to monitor him closely for problems."

D. We still need to monitor him closely for problems A baby born at 36 weeks' gestation is considered a late preterm newborn. These newborns face similar challenges as those of preterm newborns and require similar care. Telling the mother that close monitoring is necessary can prevent any misconceptions that she might have and prepare her for what might arise. The baby is not considered a term newborn, nor is the baby considered premature. The decision for care in the NICU would depend on the newborn's status. Asking the woman how she feels about the delivery demonstrates caring but does not address the woman's lack of understanding about her newborn. (ch. 23)

The nurse is describing the three phases of the uterine cycle with a client. What occurs during the proliferative phase of the uterine cycle that the nurse should include in a teaching session with the client? Answers: A. If fertilization does not occur, the corpus luteum degenerates, and hormonal levels fall. B. Withdrawal of hormones causes the endometrial cells to change, and menstruation begins. C. An ovum matures. D. While the ovarian follicles are producing increased amounts of estrogen, the endometrium prepares for possible fertilization with pronounced growth.

D. While the ovarian follicles are producing increased amounts of estrogen, the endometrium prepares for possible fertilization with pronounced growth While the ovarian follicles are producing increased amounts of estrogen, the endometrium prepares for possible fertilization with pronounced growth during the proliferative phase. If fertilization does not occur, the corpus luteum degenerates, and hormonal levels fall during the secretory phase. Withdrawal of hormones causes the endometrial cells to change, and menstruation begins. Maturation of an ovum occurs during the ovarian cycle; growth of the lining (endometrium) of the uterus occurs during the uterine cycle. Together, these cycles are known as menstrual cycle. (ch. 3)

A nurse is preparing a presentation about ways to minimize heat loss in the newborn. Which measure would the nurse include to prevent heat loss through convection? Answers: A. placing the newborn skin-to-skin with the mother B. using a radiant warmer to transport a newborn C. placing a cap on a newborn's head D. working inside an isolette as much as possible.

D. Working inside an isolette as much as possible To prevent heat loss by convection, the nurse would keep the newborn out of direct cool drafts (open doors, windows, fans, air conditioners) in the environment, work inside an isolette as much as possible and minimize opening portholes that allow cold air to flow inside, and warm any oxygen or humidified air that comes in contact with the newborn. Placing a cap on the newborn's head would help minimize heat loss through evaporation. Placing the newborn skin-to-skin with the mother helps to prevent heat loss through conduction. Using a radiant warmer to transport a newborn helps minimize heat loss through radiation. (ch. 17)

A woman is to undergo an amnioinfusion. Which statement would be most appropriate to include when teaching the woman about this procedure? Answers: A. "A suction cup is placed on your baby's head to help bring it out." B. "We'll give you an analgesic to help reduce the pain." C. "After the infusion, you'll be scheduled for a cesarean birth." D. "You'll need to stay in bed while you're having this procedure."

D. You'll need to stay in bed while you're having this procedure An amnioinfusion involves the instillation of a volume of warmed, sterile normal saline or Ringer's lactate into the uterus via an intrauterine pressure catheter. The client must remain in bed during the procedure. The use of analgesia is unrelated to this procedure. A cesarean birth is necessary only if the FHR does not improve after the amnioinfusion. Application of a suction cup to the head of the fetus refers to a vacuum-assisted birth (ch. 21)

A nurse is caring for several women in labor. The nurse determines that which woman is in the transition phase of labor? Answers: A. contractions every 21/2 minutes, cervical dilation 7 cm B. contractions every 1 minute, cervical dilation 9 cm C. contractions every 3 minutes, cervical dilation 5 cm D. contractions every 5 minutes, cervical dilation 3 cm

The transition phase is characterized by strong contractions occurring every 1 to 2 minutes and cervical dilation from 8 to 10 cm. Contractions every 5 minutes with cervical dilation of 3 cm is typical of the latent phase. Contractions every 3 minutes with cervical dilation of 5 cm and contractions every 21/2 minutes with cervical dilation of 7 cm suggest the active phase of labor. (ch. 13)

A client who is 4 months pregnant is at the prenatal clinic for her initial visit. Her history reveals she has 7-year-old twins who were born at 34 weeks gestation, a 2-year-old son born at 39 weeks gestation, and a spontaneous abortion 1 year ago at 6 weeks gestation. Using the GTPAL method, the nurse would document her obstetric history as: Answers: A. 3 1 2 2 3. B. 4 2 1 3 1. C. 4 1 1 1 3. D. 3 2 1 0 3.

c. 41113 Using the GTPAL method, the woman's history would be documented as 4 (her fourth pregnancy), 1 (number of term pregnancies), 1 (number of pregnancies ending in preterm birth), 1 (number of pregnancies ending before 20 weeks or viability), and 3 (number of living children). (ch. 12)

A postpartum client who is bottle feeding her newborn asks, "When should my period return?" Which response by the nurse would be most appropriate? Answers: A. "It's difficult to say, but it will probably return in about 2 to 3 weeks." B. "You won't have to worry about it returning for at least 3 months." C. "It varies, but you can estimate it returning in about 7 to 9 weeks." D. "You don't have to worry about that now. It'll be quite a while."

c. It varies, but you can estimate it returning in about 7 to 9 weeks For the nonlactating woman, menstruation resumes 7 to 9 weeks after giving birth, with the first cycle being anovulatory. For the lactating woman, menses can return anytime from 2 to 18 months after childbirth. (ch. 15)


Conjuntos de estudio relacionados

Quiz #7: CH.7 Long-term Memory Encoding and Retrieval

View Set

Repaso Sistemas y Ecuaciones, Ecuaciones primer y segundo grado

View Set

Chapter 2 - Nutrition Guidelines & Assessment

View Set

MGMT 4243 Summer Ch. 4-9 Quiz Questions

View Set

Wong's Ch. 6: Childhood Communicable and Infectious Diseases

View Set